Вы находитесь на странице: 1из 43

1.

A particle of mass M and charge Q moving with velocity "1 describes a circular path of radius R when subjected to a uniform transverse magnetic field of induction B. The work done by the field when the panicle completes one full circle is :

(a) (~2) 21tR (b) zero

(c) BQ 21tR Cd) BQv 2nR

2. A particle of charge - 16 x 10-'18 C moving with velocity 10 ms"! along the x-axis enters a region where a magnetic field of induction B is along the y-axis and an electric field of magnitude 104 Vim is along the negative z-axis. If the charged particle continues moving along the x-axis, the magnitude of B is :

(a) 103 Wb/m2 (b) 105 Wb/m2

(c) 1016 Wh/m2 Cd) 10-3 Wb/m2

3. A thin rectangular magnet suspended freely has a period of oscillation equal to T. Now it is broken into two equal halves (each having half of the original length) and one piece is made to oscillate freely in the same field. If its period of oscillation is T " the ratio T 'IT is :

1 . 1

(a) 2.J2 (b) ·2

(c) 2

4. A magnetic needle lying parallel to a magnetic field requires W unit of work to turn it through 60°. The torque needed to maintain the needle in this position will be :

(a)./3W (b)W

(c) C..J3/2) W Cd) 2W

5. The magnetic Jinesofforce inside a Dar magnet : (al are from north-pole to south-pole of the magnet

(b) do not exist

(c) depend upon the area of cross-section of the bar magnet

(d) aloe from south-pole to north-pole of the magnet

6. Curie temperature is the temperature above which:

(a) a ferromagnetic material becomes para magnetic

(b) a paramagnetic material becomes

diamagnetic

(c) a ferromagnetic material becomes

diamagnetic

(d) a paramagnetic material becomes

ferromagnetic

7. A spring balance is attached to the ceiling of a lift. A man bangs his bag on the spring and the spring reads 49 N, when the lift is stationary. If the lift moves downward with an acceleration of 5 m/ 82, the reading of the spring balance will be:

(a) 24 N (c) 15 N

(b) 74 N (d) 49 N

8. The length of a wire of a potentiometer is 100 em, and the emf of its stand and cell is E volt. It is employed to measure the emf of a battery whose internal resistance is O.S n. If the balance point is obtained at l = 30 em from the

positive end, the emf of the battery is : °

30E

(a) roO.S

30£ (b) 100 - '0.5

( , 30 (E - 0.5i) h o. h ° h

C) 100 ' were l IS t e current ill t e

potentiometer wire

Cd) 30E

100

9. A strip of copper and another of germanium are cooled from room temperature to 80 K. The resistance of:

(a) each of these decreases

(b) copper strip increases and that of germanium decreases

(c) copper strip decreases and that of germanium increases

(d) each of the above increases

10. Consider telecommunication through optical fibres. :which of the following statements is not true?

(a) Optical fibres can be of graded refractive index

(b) Optical fibres are subjected to electromagnetic interference from outside

(c) Optical fibres have extremely low transmission loss

(d) Optical fibres may have homogeneous core with a suitable cladding

11. The thermo-emf of a thermocouple is 25 ~ V!"C

-at room temperature. A galvanometer of.400: resistance, capable of detecting current as low as 10-5 A, is connected with the thermocouple. The smallest temperature difference that can be detected by this system is :

(a) 16°C (b) 12°C

(c) 8°C (d) 20"C

12. The negative Zn pole of Daniell cell, sending a constant current through a circuit, decreases in mass by 0.13 g in 30 minutes. If the electrochemical equivalent of Zn arid Cu are 32.5 and 31.5 respectively, the increase in the mass of the positive Cu pole in this time is :

(a) 0.180 g (b) 0.141 g

ec) 0.126 g Cd) 0.242 g

13. Dimensions of_1_, where symbols have their f-lQ So

usual meaning, are ; (a) [L-1T]

ec) [l}T-2]

(b) [L2T2] Cd)[LT-1)

14. A circular disc X of radius R is made from an iron plate of thickness t, and another disc Y of radius 4R is made from an iron plate of thlcknessz/a . Then the relation between the moment of inertia I x and 1 y is :

(a) Iy = 32Ix . (b) Iy "" 16Ix

(c) Iy = l x (d) Iy = 64lx

15. The time period of a satellite of earth is 5 hours.

If the separation between the earth and the satellite is increased to 4 times the previous value, the new time petiod will become:

(a) 10 h (b) 80 h

(c) 40 h Cd) 20 h

16. A particle performing uniform circular motion has angular momentum L. If its angular frequency is doubled and its kinetic energy halved, then the new angular momentum is ;

L (a) - 4

(c) 4L

(b) 2L Cd) _!,_ 2

"

17. Which of the follOwing radiations has the least wavelength?

(a) y-rays (b) ll"-rays

(c) a-rays Cd) X·rays

18. When U23S nucleus originally at rest, decays py .: emitting an alpha particle having a speed u, the recoil speed of the residual nucleus is :

4u 4u

(a) 238 (b) - 234

4u 4u

(e) 234 Cd) - 238

19. Two spherical bodies of mass M and SM and radii Rand 2R respectively are released in free space with initial separation between their centres equal to 12R. If they attract each other due to gravitational force only, then the distance covered by the smaller body JUSt before collision

is :

(a) 2.SR (c) 7.SR

(b) 4.SR Cd) 1.SR

20. The difference in with temperature semiconductor arises difference .in the:

(a) crystal structure

(b) variation of the number of charge carriers with temperature

(c) type of bonding

(d) variation of scattering mechanism with temperature

21. A car moving/with a speed of 50 km/h, can be stopped by brakes after at least 6 rn, If the same car is moving at a speed of 100 km/h, the minimum stopping distance is :

(a) 12 m (b) 18 m

(c) 24 m (d) 6 m

22. A boy playing on the roof of a 10 m high building throws a ball with a speed of 10 m/s at an angle of 30° with the horizontal, How far from the throwing point will the ball be at the height of 10 m from the ground? [g == 10 m/s2, sin 30° = 1/2,

cos 30" = .J312]

(a) 5.20 m (b) 4.33 m

(c) 2.60 m (d) 8.66 m

23. An ammeter reads upto lAo Its Internal resistance is 0.81 n. To increase the range to 10 A, the value of the required shunt is :

the variation of resistance in a metal and a essentially due to die

(a) 0.03.0 (c) 0.9n

(b) 0.30 Cd) 0.090

24. The physical quantities not having same dimensions are:

(a) torque and work

(b) momentum and Planck's constant ec) Stress and Young's modulus

Cd) speed and (!J. oEorll2

25. Three forces stan acting C simultaneously oil a panicle moving with

velocity 1. These forces are represented in magnitude and direction

by the three sides of a A B triangle ABC (as shown). The particle will now move with velocity:

(a) less than 1

-+ (b) greater than v

(c) lv I in the direction of largest force Be Cd) It, remaining unchanged

26. If the electric flux entering and leaving an enclosed surface respectively is $1 and ~2' the electric charge inside the surface will be ;

(a) (lPz - +I)eo (b) (+1 + '2)/"0

eC) (92 - 91)/"0 (d) (~l + h) eo

27. A horizontal force of F

10 N is necessary to

JUSt hold a block _~l~ON~jllll stationary against a

wall. The coefficient of friction between the block and the wall is

0.2. The weight of the block is :

(a) 20 N (b) 50 N

ec) 100 N Cd) 2 N

28. A marble block of mass 2 kg lying on ice when given a velocity of 6 m/s is stopped by friction in 10 s. Then the coefficient of friction is ;

(a) 0.02 Cb) 0.03

(c) 0.06 (d) 0.01

29. Consider the following two statements :

A. Linear momentum of a system of particles is zero.

B. Kinetic energy of a system of particles is zero.

Then;

(a) A does not imply B and B does not imply A (b) A implies B butB does not imply A

(c) A does not imply B but B implies A

(d) A implies Band B implies A

30. Two coils are placed close to each other. The mutual inductance of the pair of coils depends upon:

(a) the rates at which currents are changing in the tWO coils

(b) relative position and orientation of the two coils

ec) the materials of the wires of the coils (d) the currents in the two coils

31. A block of mass M is pulled along a horizontal frictionless surface by a . rope of mass m. If a force P is applied at the free end of the rope, the force exerted by the rope on the block.is :

() Pm (b) Pm

a M+m M-m

(c)P (d)~

M+m

32. A light spring balance hangs from the hook of the other light spring balance and a block of mass M kg hangs from the former one. Then the true statement about the scale reading is :

(a) both the scales read M kg each

(b) the scale of the lower one reads M kg and of the upper one zero

(c) the reading of the two scales can be anything but the sum of the readings will be Mkg

(d) both the scales 'read M/2 kg

33. A wire suspended vertically from one of its ends is stretched by attaching a weight of 200 N to the lower end. The weight stretches the wire by 1 nun. Then the elastic energy stored in the wire is :

(a) 0.2 J " (b) 10 J

(c) 20 J Cd) 0.1 J

34. The escape velocity for a body projected vertically upwards from the surface of earth is 11 km/s. If the body is projected at an angle of 45" with the vertical, the escape velocity will be:

(a) 11 .J2 km/s (b) 22 krn/s

(c) 11 km/s Cd) 11/.J2 m/s

35. A mass M is suspended from a spring of negligible mass: The spring is pulled a little and then released so that the mass executes SHM of time period T. If the mass is increased by m, the

time period becomes 5T/3, then the ratio of r; is :

(a) ~ (b) 2S

5 9

(c) 16 (d) ~

9 3

- 36. ~Heat cannot be itself flow from a body at lower temperature [0 a body at higher temperature" is a statement or consequence of:

(a) second law of thermodynamics

(b) conservation of momentum

(c) conservation of mass

(d) first law of thermodynamics

37. Two particles A and B of equal masses are suspended from two massless springs of spring constants kl and k2' respectively. If the maximum velocities, during oscillations are equal, the ratio of amplitudes of A and B is :

(a) Jkl/k2 (b) kl/k2

(c) Jk21kl (d) kz/ kl

38. The length of a simple pendulum executing simple harmonic motion is increased by 21%. The percentage increase in the time period of the pendulum of increased length is:

(a) 11% (b) 21%

(c) 42% (d) 10.5%

39. The displacement y of a wave travelling in the x-direction is given by

y 0; 1O~4 sin (600t ~ 2x ;- -j) metre,

where, x is expressed in metres and t in seconds. The speed of the wave-motion, in ms'" is:

(a) 300 (c) 1200

(b) 600 (d) 200

40. When the current changes from + 2 A to - 2A in 0.05 s, an emf of 8 V is induced in a coil. The coefficient of self- induction of the coil is :

(a) 0.2 H (c) 0.8 H

(b) 0.4 H (d) 0.1 H

41. In an oscillating Le circuit the maximum charge on the capacitor is Q. The charge on the capacitor when the energy is stored equally between the electric and magnetic fields is :

(a) Q 12 (b) Q 1.J3

(c) Q 1../2 Cd) Q

42. The core of any transformer is laminated so as to:

(a) reduce the energy loss due to eddy currents (b) make it light weight

(c) make it robust and strong

(d) increase the secondary voltage ----)

· .. 43. Let P be the force acting ona particle having

position vector "1 and i be the torque of this force about the origin. Then:

----)----) ~4

(a) r . t ::: 0 and F . t ~ 0

44 4------}

(b) r . t ;t 0 and F .. t ::: 0

~-----}o -----}o~

(c) r .. t ~ 0 and F . t ~ 0

44 4---).

(d) r . t == 0 and F . 'C "" 0

44. A radioactive sample at any instant has its disintegration rate 5000 disintegrations per minute. After 5 minutes, the rate is 1250 disintegrations per minute. Then, the decay constant (per minute) is:

(a) 0.4 In 2 (b) 0.2 In 2

(c) 0.1 In 2 Cd) 0.8 In 2

45. A nucleus with Z "" 92 emits the following in a

sequence: 0:, 0:, .13~, Ir, 0:, 0:,0:,0:; 13-, ~~,

0:, 13+, W, Cl".. The Z of the resulting nucleus is :

Ca) 76 (b) 78

l:~!'~'

(e) 82 ~.i.. (d) 74

46. TWo identical, photocathodes receive light of frequencies 11. and f2• If the velocities of the photoelectrons (of mass m) coming out are respectively v1. and v2, then:

2 . 2 2h .

. (a) VI - V2 = - (iI - '2)

. . m

[2h ]112

(b) VI + V2 '" 1TI (fI + '2)

2 2 2h

(c) Vt + V2 ::: m (II + h)

[2h ]112

(d) VI - V2 == m (11 - (2)

47. Which of the following cannot be emitted by radioactive substances during their decay?

(a) Protons (b) Neutrinos

(c) Helium nuclei (d) Electrons

48. A 3V battery with negligible internal resistance is connected in a circuit as shown in the figure. The current 1, in the circuit will be :

·1

3V

(a) 1 A (c) 2A

3n (b) 1.5 A

Cd) ~A

49. A sheet of alurnini urn foil of negligible thickness is introduced between the plates of a capacitor. The capacitance of the capacitor:

(a) decreases (b) remains unchanged

(c) becomes infinite (d) increases

50. The displacement of a particle varies according co the relation x'" 4 (cosx; + sin 1fl'). The amplitude of the particle is :

(a) - 4 (b) 4

(c) 4 ./2. Cd) 8

51. A thin spherical conducting shell of radius R has a charge q. Another charge Q is placed at the centre of the shell. The electrostatic potential at a point P at a distance R/2 from the centre of the shell is:

2Q (a) 41lf-oR

(c) 2Q +_q_

41leoR 41ltuR

52. The work done in placing a charge of ? x 10- UI C on a condenser of capacity 100 1-1. F

is:

(a) 16 xlO-32 J (c) 4 x 10-10 J

(b) 3.1 x 10-26 J (d) 32 x 10-32 J

53. The co-ordinates of a moving particle at any time t are given by :x :=: at 3 and y '" (lr 3. The

speed of the panicle at time t is given by ;

(a) 3t Ju2 .,. ~/- (b) 3[2 Ju2 + ~i

(e) [2 ~U2 + p2 (d) Ju2 + (32

54. During an adiabatic process, the pressure of a gas is found to be proportional to [he cube' of its absolute temperature .. The ratio C p lev for the gas is:

(3) 4/3 (e) 5/3

(b) 2 (d) 3/2

55- Which of the following parameters does not characterise the thermodynamic state of matter?

(a) Temperature (b) Pressure

(e) Work (d) Volume

56. A Carnot enginetakes 3 x 106 cal of heat from a reservoir at 627°C and gives it to a sink. at.2 7°C. The work done by the engine is :

(a) 4.2 x 106 J (b) 8.4 x 106 J

(c) 16.8 x 106 J (d) zero

57. A spring of spring constant 5 x 103 N/m is stretched initially by 5 em from the unstretched position. Then the work required to stretch it

further by another 5 em is : -

(a) 12.50 N-m (b) 18.75 N·m

ee) 25.00 N-m (d) 6.25 N-m

58. A metal wire of linear mass density of 9.8 g/rn is stretched with a tension of 10 kg-wt between two rigid supports 1 m apart. The wire passes at its middle point between the poles of a permanent magnet and it vibrates in resonance when carrying an alternating current of frequency n. The frequency n of the alternating source is : (a) 50 Hz ec) 200 Hz

(b) 100 Hz (d) 25 Hz.

59. A tuning fork of known frequency 256 Hz makes 5 beats/s with the vibrating string of a piano. The beat frequency decreases [Q 2 beats!s when the tension in the piano string is slightly increased. The frequency of the piano string before increasing the tension was :

(a) (256 + 2) Hz. (b) (256 - 2) Hz

(c) (256 - 5) Hz (d) (256'1- 5) Hz

60. A body executes simple harmonic motion. The potential energy (PE), the kinetic energy eKE) and total energy (TE) are measured as function of displacement x. Which of the following statement is true?

(a) KE is maximum when x :. 0 (b) TE is zero when x == 0

(c) KE is maximum when x is maximum (d) PE is maximum when x = 0

61.ln the nuclear fusion reaction,

fH +rH __".1He +11

given that the repulsive potential energy between the two nuclei is 7.7 x IO-·14J, the temperature at which the gases must be heated to initiate the reaction is nearly [Boitl.mann's constant k == 1.38 x 10-23 J/K ] :

(a) 107 K (b) lOS K

ee) 101 K Cd) 10') K

62. Which of rhe following atoms has the lowest ionization potential?

(a) Ii N (b) 1~%Cs

(c)i~Ar Cd) 19O

63 .. The wavelengths involved in "the spectrum of deuterium CiD} are Slightly different from that

of hydrogen. spectrum, because:

(a) sizes of the two nuclei are different

(b) nuclear forces are djfferent in the two cases (c) masses of the two nuclei are different

(d) attraction between the electron and the nucleus is different in the two cases

64. In the middle of the depletion layer 01 reverse biased p·n junction, the:

(a) electric field is zero

(b) potential is maximum

(c) electric field is maximum (d) potential is zero

(a) q2 - q3 cos 8 (b)q~ ... qJ sin 6

b2 a2 b a

(c) q2 + q;J cos 0 (d) q2 - q3 sin e

b2 a2 b2 a2

70. A 220 V, 1000 W burt! is connected across a 110 V mains supply. The power consumed will be :

(a) 750 W (b) 500 W '

(c) 250 W (d) 1000 W

71. The image formed by an objective of a compound microscope is:

(a) virtual and diminished

(b) real and diminished

(e) real and enlarged

(d) virtual and enlarged

72. The earth radiates in the infra-red region of the spectrum. The spectrum is correctly given by: (a) Rayleigh Jeans law

(b) Planck's law of radiation (c) Stefan's law of radiation (d) Wien's law

73. To get three images of a single object, one should have two plane mirrors at an angle of :

(a) 600 (b) 90°

(c) 1200 (d) 300

74. According to Newton's law of cooling, the rate of cooling of a body is proportional to (6 8)", where

6, 0 is the difference of the temperature of the body and the surroundings, and n is equal to ;

(a) 2 (b) 3

(c) 4 (d) 1

75. The length of a given cylindrical wire is increased by 100%. Due to the consequent decrease in diameter the change in the resistance of the wire will be :

(a) 200% (b) 100%

(c) 50% (d) 300%

...eIII\~ __

65 .. l.f the binding energy of the electron in a hydrogen atom is 13.6 eV, the energy required to remove the electron from the first excited state of Li2+ is :

(a) 30.6 eV (b) 13.6 eV

(e) 3.4 eV Cd) 12.2.4 eV

66. A body is moved along a straight line by a machine delivering a constant power. The distance moved by the body in rime r is proportional to :

(a) t 3/4 (b) t312

(c) tli~ (d) tin

67. A rocket with a lift-off mass 3.5 x 104 kg is blasted upwards with an initial acceleration of 10 m/82 . Then the initial thrust of the blast is :

(a) 3.5 x lOS N (b) 7,Ox lOS N

(c) 14.0 X 105 N «n 1.75 x lOs N

68. To demonstrate the phenomenon of

interference we require two sources which emit radiations of :

(a) nearly the same frequency (b) the same frequency

(c) different wavelength

(d) the same frequency and having a definite phase relationship

69. Three charges - q l' + q 2 and - q3 are placed as shown in the figure. The x-component of the force on - ql is proportional to :

~!l3 y

~

II

~·~b

~ql

76. In Bohr series of lines of hydrogen spectrum, the third line from the red end corresponds to which one of the following inner-orbit jumps of the electron for Bohr orbits in an atom of hydrogen? (a) 3-,1> 2 (c) 4~ 1

(b).5~ 2 (d) 2-+ 5

77. The de-Broglie wavelength of a tennis ball of mass 60g moving with a velocity of 10 m/s is approximately:

(Planck's constant, h = 6.63 X 10-34 Js)

(a) 10-33 m (b) 10-31 m

(c) 10 .. 16 rn (d) 1O~2s m

78. The orbital angular momentum for anelecrron revolving in an orbit is given by JI(l + 1) !.

This momentum for an s-elecrron will be given by:

1 h (a) + -:-.- 2 2n

h (e)-

21t

(b) zero

r;:; h (d) 'oJ L.' .- 2n

79. How many unit cells are present in a cube shaped. idea] crystal of NaC! of mass LOO g? [Atomic masses : Na "" 23, Cl = 35.5]

Ca) 2.57 x 1021 (b) 5.14 x 1021

(c) 1.28 x 1021 (d) 1.7l x 1021

80. Glass is a :

(a) micro-crystalline solid (b) super-cooled liquid (c) gel

(d) polymeric mixture

81. Which one of the following statements is correct?

(a) Manganese salts give a violet borax- bead test in the reducing flame

(b) From a mixed precipitate of AgCl and AgI, ammonia solution dissolves only AgCl

(c) Ferric ions give a deep green precipitate on adding potassium ferro cyanide solution

(d) On boiling a solution having K+, Ca2+ and HCO i ions we get a precipitate of K2Ca(C03)2

82. According to the periodic law of elements, the variation in properties of elements is related to their:

(a) atomic masses (b) nuclear masses (c) atomic numbers

(d) nuclear neutron-proton number ratios

83. Graphite is a soft solid lubricant extremely difficult to melt. The reason for this anomalous behaviour is that graphite:

(a) is a non-crystalline substance (b) is an allotropic form of diamond

(c) has molecules of variable. molecular masses like polymers

(d) has carbon atoms arranged in farge plates of rings of strongly bound carbon atoms with weak interplate bonds

84. The IUPAC name ofCH3COCH(CH3h is : (a) isopropylrnethyl ketone

(1)) 2·methyJ·3-butanone

(c) 4-methylisopropyJ ketone (d) 3-methyl-2-butanone

85. When CH2 =CH-COOH is reduced with LiAIH4' the compound obtained will be :

(a) CH3 -OI2 -COOH

(b) CH2 =CH-CH20H

(c) CH3 -CH2 -CH20H

(d) CH3 -CH2 -CHO

86. According to the kinetic theory of gases, in an ideal gas, between two successive collisions a ga1 molecule travels :

(a) in a circular path

(b) in a wavy path

(c) in a straight line path

Cd) with an accelerated velocity

87, Which of the following group of transition metals is called coinage metals? .

(a) Cu, Ag, Au (b) Ru, Rh, pd (c) Fe, Co, Ni (d) Os, Ir, Pt

88. The general formula C "Hz"o z could be for open chain:

(a) diketones

(b) carboxylic acids (c) diols

(d) dialdehyde.s

89. An ether is more volatile than an alcohol having the same molecular formula. This is due to : (a) dipolar character of ethers

(b) alcohols having resonance structures

(c) inter-molecular hydrogen bonding in ethers (d) inter-molecular hydrogen bonding in

alcohols

90. Among the following four structures I to IV :

CH3 0 CH3

I II I

C2HS -CH-C3~ C}h-C -CH-C2HS

0) OQ

CH3

I

C2HS -CH-C2HS

H

I

H-C€I

I H (Ill)

(IV)

it is true that:

(a) all four are chiral compounds

(b) only I and II are chiral compounds (c) only III is a chiral compound

(d) only II and IV are chiral compounds

91. Which one of the following processes will produce hard water?

(a) Saturation of water with CaC03 (b) Saturation of water with MgC03 (c) Saturation of water with CaS04 (d) Addition of Na2S04 to water

92. Which one of the following compounds has the smallest bond angle in its molecule 7

(a) S02 (b) OH2

(c) SH2 (d) NH3

93. Which one of the following pairs of molecules will have permanent dipole moments for both .rnembers ?

(a) SiF4 and N02 (b) N02 andCOz

(c) N02 and 03 (d) SiF4 and CO2

94. Which one of the following groupings represents a collection of isoelectronic species? (At. numbers: Cs·SS, Br·3S)

(a) Na+, Ca2+, Mg2+ (b) N3-, P-, Na+

(c) Be, AIH, cr (d) Ca2+, Cs+, Br

95.ln the anion HCOO- the two (arbon-oxygen bonds are found to be of equal length, What is the. reason for it ?

(a) Electronic orbits of carbon atom are hybridised

(b) The C = ° bond is weaker than the C -0 bond

(c) The anion i-ICOO- has two resonating structures

(d) The anion is obtained by removal of a proton from the acid molecule

96. The pair of species having identical shapes for molecules of both species is ~

(a) CF4, SF4 (b) XeF2, CO2

.. (c) Up:" PC13 (d) PFs, IPs

97. The atomic numbers of vanadium (V) , chromium (Cr), manganese (Mn) and iron (Fe) are respectively 23,.24,25 and 26. Which one of these may beexpeeted to have the highest second ionization enthalpy?

(a) V (b) Cr

(c) Mn (d) Fe

98. Consider the reaction equilibrium; 2S02(g)+ 02(g)~ 2S03(g);

tI HO",-I98 kJ On the basis of Le-Chatelier's principle, the condition favourable for the forward reaction is : (a) lowering of temperature as well as pressure (b) increasing temperature as well as pressure (c) lowering the' temperature and increasing

the pressure

(d) any va lue of temperature and pressure

99. What volume of hydrogen gas, at 273K and 1 atm pressure will be consumed in obtaining 21.6g of elemental boron (atomic. mass = 10.8) from the reduction of boron trichloride by hydrogen? (a) 89.6 L (c) 44.8 L

eb) 67.2 L (d) 22.41.

100. For the reaction equilibrium,

Np 4(g)~· 2NOCg)

the concenrrations of N204 and N02 at equilibrium are 4.8 x 10-2 and 1.2x 10-2 mol L-1 respectively. The value of x, for the reaction is ;

(a) 3.3)( 102 mol r;l (b) 3)( 10-.1 mol L-1

ee) 3 x 10-3 mol L-) (d) 3 x 103 mol L-1

101. The solubility in water of a sparingly soluble salt AB2 is 1.0 x 10-5 mol 1-1. Its solubility product

number will be : (a) 4 x 1O-1S

(c) 1 x 10-15

(b) 4)( 10-10 Cd) 1 x 10-10

102. When during electrolysis of a sqlution of AgN03, 9650 coulombs of charge pass through the electroplating bath, the mass of silver deposited on the cathode will be ;

(a) 1.08 g (b) 10,8 g

(c) 21.6 g (d) 108 g

103. For the redox reaction

2n(s)+ Cu2+ (0.1 M),. 2n2+ (1M) + Cu(s)

taking place in a cell, E '~II is 1.10 volt. Hoell for thecell will be : ( 2.303 7' '" 0.0591)

(a) 2.14 V (b) LBO V

(c) l.07 V (d) 0.82 V

104. In a 0.2 molal aqueous solution of a weak acid HX, the degree of ionisation is 0.3. Taking Kf for water as 1.85, the freezing point of the solution will be nearest to :

(a) - 0.480°C (c) - O.260°C

(b) - 0.360" C (d) + 0.480°C

105. The rare law for a reaction between the substances A and B is given by rate '" k [ At [B)m. On doubling the concentration

of A and halving the concentration of B, the ratio of the new rate to the earlier rate of the reaction will be as :

1

(a) 2m., ~. (b) em + n)

(c) (n - m) (d) 2(" - m)

106.25mL of a solution of barium hydroxide on titration with 0.1 molar solution of hydrochloric acid gave a titre value of 35 mL. The molarity of barium hydroxide solution was :

(a) 0.07 (b) 0 . .14

(e) 0.28 (d) 0.35

107. The correct relationship between free energy change in a reaction and the corresponding equilibrium constant K, is :

(a) IlG '" RT In Kc (b) - t.G '" RT In KG

(c) AGo", RT In Kc (d) - /lGo "" RT In K,

108. If at 298K the bond energies of C--l-I, C~C.

C = G and Ii-H bonds are respectively 414, 347, 615 and 435 k J mol:", the value of enthalpy change for the reaction

HlC - CHz{g) + H2(g) ----)0 H3C - CH3(g)

at 298K will be :

(a) +- 250 kJ (e) + 125 k J

(b) - 250 kJ (d) -12SkJ

~09. The enthalpy change for a reaction does not depend upon the:

(a) physical state of reactants and products

(b) use of different reactants for the same product

(c) nature of intermediate reaction steps

Cd) difference in initial or final temperatures of involved substances

110. A pressure cooker reduces cooking time for food because:

(a) heat is more evenly distributed in the cooking space

(b) boiling point of water involved in cooking is increased

(c) the higher pressure inside the cooker crushes the food material

(d) cooking involves chemical changes helped by a rise in temperature

111. If liquids A and B form an ideal solution, the; (a) enthalpy of mixing is zero

(b) entropy of mixing is zero

(c) free energy of mixing is zero

Cd), free energy as well as the entropy of mixing are each zero

112. For the reaction system;

2NO(g) + 02(g) ---+ 2N02(g) volume is suddenly reduced to half its value by increasing the pressure on it. If the reaction is of first order with respect to O2 and second order with respect to NO; the rate of reaction will : (a) diminish to one-fourth of its initial value (b) diminish to one-eighth of its initial value (c) increase to eight times of its initial value (d) increase to four times of its initial value

113. For a eel! reaction involving a two-electron change, the standard emf of the cell is found to be 0.295 V at 25°C. The equilibrium constant of the reaction at 25°C will be :

(a) 1 x 10-10 (b) 29.S x 10-2

(c) 10 Cd) 1 x 1010

,114;ln an irreversible process taking place at constant T and P and in which only pressure-volume work is being done, the change in Gibbs free energy (dG) and change in entropy (ciS), satisfy the criteria:

(a) (dS)v, Ii < 0, (dG).r, p < 0

(b) (dS)v,£ > 0, (dG)l'.p < 0

(c) (dS)v. E == 0, (dG~. P == 0

(d) (dS)v, E = 0, (dG)T, P > 0

115. Which cine of the following characteristics is not correct for physical adsorption?

(a) Adsorption on solids is reversible

(b) Adsorption increases with increase in

temperature

(c) Adsorption is spontaneous

(d) Both enthalpy and entropy of adsorption are negative

116.ln the respect of the equation k '" Ae-E"IRT in chemical kinetics, which one of the following statements is correct?

(a) k is equilibrium constant (b) A is adsorption factor

ec) E« is energy of activation (d) R is Rydberg constant

117. Standard reduction electrode potentials of three metals A, Band C are+ O.5V, - 3.0Vand -1.2V respectively. The reducing power of these metals are:

(a) B >C > A (c) C > B > A

(b) A >B >C Cd) A>C >B

118. Which one of the following substances has the highest proton affinity ?

(a) H20 (b) H2S

(c) NH3 Cd) PH3

119. Which one of the following is an amphoteric oxide?

(a) ZnO Cb) Nap

(c) S02 (d) BP3

120. A red solid is insoluble in water. However it becomes soluble if some KI is added to water. Heating the red solid in a test tube results in liberation of some violet coloured fumes and droplets of a metal appear on the cooler parts of the test tube. The red solid is :

(a) (NH4hCr20", (b) Hg11

ec) HgO Cd) Pb304

121. Concentrated hydrochloric acid when kept in open air sometimes produces a cloud of white fumes. the explanation for it is that:

(a) concentrated hydrochloric acid emits strongly smelling HCI gas all the time

(b) oxygen in air reacts wtih the emitted HCI gas to form a cloudof chlorine gas

(c) strong affinity of HCl gas for moisture in air results in forming of droplets of liquid solution which appears like a cloudy smoke

(d) due to strong affinity for water, concentrated hydrochloric add pulls moisture of air towards itself. This moisture forms droplets of'water and hence thedS{ud

122. The substance used in Holmes signals of the ship is a mixture of:

(a) CaC:.! + Ca3P2 (b) Ca3(P04)2 + PbP4

(c) HjP04 + CaCl2 Cd) NHJ + HOC!

123. The number of d-electrons retained in Fe2+ (At. no. Fe == 26) ions is :

~a) 3 (b) 4

(e) 5 (d) 6

124. What would happen when a solution of potassium chromate is treated with an excess of dilute nitric acid?

(a) C1.3+ and Crpr are formed (b) Crp;~ and H20 are formed

(c) crO~- is reduced to + 3 state of Cr (d) None of the above

125. In the co-ordination compound, l(4[NiCCN)4]' the oxidation state of nickel is :

(a) -1 (b) 0

(c)+l (d)+2-

126. Ammonia forms the complex ion [Cu(NH3 )4] 2 ... with copper ions in the alkaline solutions but not in acidic solutions. What is the reason for it ? (a) In acidic solutions hydration protects

copper ions

(b) In acidic solutions protons co-ordinate with ammonia molecules forming NH~ ions and

NH3 molecules are not available

(c) In alkaline solutions insoluble Cu{OHh is precipitated which is soluble in excess of any alkali

(d) Copper hydroxide is an amphoteric substance

127. One mole of the 'complex compound Co(NH3 hCls, gives 3 moles of ions on' dissolution in water. One mole of the same complex reacts with twO moles of AgN03 solution to yield two moles of AgCI(s). The structureof the complex is :

(a) [Co(NH3 )5Cl]CI2

(b) [Co(NH3hCI2)· 2NH3 (c) [Co(NH3 ).; CJ:iJCI· NHs Cd) [Co(NH3)4C1]Clz ·NH3

128. The radius of La 3+ (Atomic number of La == 57) is 1.06 A. Which one of the following given values will be closest to the radius of Lu3., (Atomic number ofLu == 7l)?

(a) 1.60 A (b) 1.40 A

(c) 1.06 A Cd) 0.85 A

129. The radionudide 2~~Th undergoes two successive p-decays followed by one a-decay. The atomic number and the mass number respectively of the resulting radionudide are:

(a) 92 and 234 (b) 94 and 230

(c) 90 and 230 (d) 92 and 230

130. The half-life of a radioactive isotope is 3 h. If the initial mass ofthe isotope were 256 g, the mass - of it remaining undecayed after 18 h would be :

(a) 4.0 g (b) 8.0 g

(c) 12.0 g Cd) ]6.0 g

131. Several blocks of magnesium are fixed to the bottom of a ship to :

(a) keep away the sharks (b) make the ship lighter

(c) prevent action of water and salt

(d) prevent puncturing by under-sea rocks

132. In curing cement plasters water is sprinkled from time to time. This helps in :

(a) keeping it cool

(0) developing interlocking needle- like crystals of hydrated silicates

(c) hydrating sand and gravel mixed with cement

(d) converting sand into silicic acid

133. Which one of the following statements is not true?

(a) The conjugate base ofH2PO;; is HPO~(b) pH + pOH == 14 for all aqueous solutions (c) The pH ofl x 10-~ M Hel is 8

(d) 96,500 coulombs of electricity when passed through a CuSO 4 solution deposit 19 equivalent of copper at the cathode

134. The correct order of increasing basic nature for the bases NH~, CH)NH2 and (CH3hNH is:

(a) CH3NH2 < NH3 < (CH3)zNH

(b) (CH3)2NH < NH3 < CJ-I~NH2

(c) NH_., < CH3NH2 < (CH3)2NH

Cd) CH1NH2 < (CH"hNH < NH3

135. Butene-l may be converted to butane by reaction with :

(a) Zn-HCI (b) Sn-HC\

(c) Zn-Hg (d) PdIH2

136. The solubilities of carbonates decrease down tile magnesium group due to a decrease in .:

(a) lattice energies of solids

(b) hydration energies of cations (c) inter-ionic attraction

(d) entropy of solution formation

137. During dehydration of alcohols to alkenes by heating with concentrated H2SO 4 the initiation step is :

(a) protonation of alcohol molecule (b) formation of carbocatlon

(c) elimination of water

(d) formation of an ester

138. Which one of the following nitrates win leave behind a metal Oil strong heating?

(a) Ferric nitrate (b) Copper nitrate

(c) Manganese nitrate(d) Silver nitrate

139. When rain is accompanied by a

thunderstorm, the collected rain water will have a pH value:

(a) slightly lower than that of rain water without thunderstorm

(b) slightly higher than that when the thunderstorm is not there

(c) uninfluenced by occurrence of

thunderstorm

(d) which depends on the amount of dust in air

140. Complete hydrolysis of cellulose gives :.

(a) D·fructose (b) D·ribose

(c) D-glucose (d) L-glucose

141. FOf making good quality mirrors, plates of float glass are used. These are obtained by floating molten glass over a liquid metal which does not . solidify before glass. The metal used can be :

(a) mercury (b) tin

(e) sodium Cd) magnesium

142. The substance nat likely to contain CaC03 is : (a) a marble statue (b) calcined gypsum

(c) sea shells (d) dolomite

143. The reason far double helical structure of DNA is operation of :

(a) van der waals' forces·

_ (b) dipole-dipole interaction (e) hydrogen bonding

(d) electrostatic attractions

144. Bottles containing C6HsI and C6HsCR2i lost their original labels. They were labelled A and B for testing. Aand B were separately taken in a test tube and boiled with NaOH

, solution. The end solution in each tube was made acidic with dilute HN03 and then some AgN03 solution was added. Substance B gave a yellow precipitate. Which one of the following statements is true far this experiment ?

(a) A was C6HsI

(b) A wasC6HsCH21 (c) B was C6HSI

Cd) Addition ofHN03 was unnece~sary

145. Ethyl isocyanide on hyd rolysis in acidic medium generates;

(a) ethylarnine salt and merhanolc acid (b) propanoic add and ammonium salt (c) ethanoic add and ammonium salt (d) methylamine salt and erbanoic acid

146. The internal energy change when a system goes from state A to B is4D kJ/mol. If the system goes from A to B bya reversible path and_ .. returns to state A by an irreversible path, what would be the net change in internal energy ?

(a) 40 kJ (b) > 40 kJ

(c) < 40 kJ (d) zero

147. The reaction of chloroform with alcoholic KOH and p-toluidine form :

(a) H3C-@-CN

(b) H3C-@-N2Cl'

(e) H3C-@-NHCHC12 Cd) H3C-@-NC

148. Nylon threads are made of: (a) polyvinyl polymer

(b) polyester polymer

(c) polyamide polymer

(d) polyethylene polymer

149. On mixing a certain alkane with chlorine and irradiating it with ultraviolet light, it forms only one monochloroalkane. This alkane could be;

(a) propane (b) pentane

(e) isopentane (d) neopentane

150. Which of the following could act as a propellant for rockets ?

(a) Liquid hydrogen + liquid nitrogen (b) Liquid oxygen + liquid argon

(c) Liquid hydrogen + liquid oxygen (d) Liquid nitrogen + liquid oxygen

~------------------

1. A function f from the set of natural numbers to integers defined by

{O-l h . dd

-y,wennlso

f(n) =

-%, when n is even

is:

(a) one-one but not onto

(b) onto but not one-one

(c) one-one and onto both (d) neither one-one nor onto

2. Let ZI and z2. be two roots of the equation Z2 +.az + b = 0, zheing complex. Further,

assume that the origin, Zl and Z2 form an equilateral triangle. Then:

(a) ci = b (h) a2 = 2b

(c) a2 = 3b (d) al = 4b

3. If z and 00 are two non-zero complex numbers such that] z roj = 1,. and

arg (z) - arg (00) = ~, then zro is equal to :

(a) 1 (b)-1

(e)i (d)-i

(1 + i)X

4- If 1 _ i = 1, then:

(a) x '" 4 0, where n is any positive integer (b) x", 211, where n is any positive integer

(c) x = 4 n + 1, where 11 is any positive integer . (d) x = 2 n 'I- 1, where n is any positive integer

a el- 1'1- a3
5. If b b2 1-1' b3 ",0
c e2 1 + ,3 and vectors (1, a, a2), (1, b, b2) and (1, C, (2) are non-coplanar, then the product abc equals :

(a) 2 (b) -1

(c) 1 (d) 0

6. If the system of linear equations 'x+2ay+az=0

x +3by + bz = 0 x+4cy+cz=0

has a non-zero solution, then a, b, c : (a) are in AP

(b) are in GP

ee) are in HP

(d) satlsfy c « 2b+ 3c=0

7. If the sum of the roots of the quadratic equation ax2 + bx + C = 0 is equal to the sum of the

squares of their reciprocals, then E:, !?_ and -be. are c a

in:

Cal arithmetic progression (b) geometric progression ec) harmonic progression

(d) anthmetico-geornerric progression

8. The number of the real solutions of the equation Xl - 3 J x I + 2 = 0 is :

(a) 2 _ ec) 1

(b) 4 (d) 3

9. The value of 'a' for which one root of the quadratic equation

Ca2 - Sa + 3) xl + (3a -1) x + 2= a is twice as large as the other, is ;

(a) 2/3 (b) -2/3

(c) 1/3 (d) -1/3

10. If A 0=[: . :] and A2 0= [~ :} then:

(a) a '" a2 + b2, ~:::: ab

(b) a =a2 + bZ, 13= 2ab (c) a "" a2 + b2, f3. '" 02 - b2 (d) ex = 2ab, 13 "" 02 + b2

11, A student is to answer 10 out of 13 questions in an examination such that he. must choose at least 4 from the first five questions. The number of choices. available to him is :

(a) 140 (c) 280

(b) 196 Cd) 346

12. The number of ways in which 6 men and 5 women can dine at a round table, if no two women are to. sit together, is given by : .

(a)6!x5! (b) 30

(c) 5!)(41 (d)7!.x5!

13. If 1, ro, ro2 .are the cube roots of unity, then :

1 ro" a/"
8= OJ" OJ2n 1
ro2n 1 w" is equal to : (a) 0

(c) CO

(b) 1 (d) (fJ2

14. If "C, denotes the number of combinations of n things taken r at a time, then the expression -c, + J + "C, -1 + 2 X ''C,equals :

(a) n+ -c, (b)" + -c, + 1

(e) n e- 'c, (d) , .... 'c, >- I

15. The number of integral terms in the expansion of (-,J3 + VsiS6 is :

(a) 32 (b) 33

(e) 34 (d) 35

16. If x is positive, the first negative term in the expansion of (1 + xi7.!s is :

(a) 7th term (b) 5th term

ee) 8th term (d) 6th term

17. The sum of the series

1 1 1 . I

1-2 - 2.:3 + T4 - ... upto 0) IS equa to:

(a) 210g< 2 (b) loge 2 - 1

(c) log. 2 (d) log. (~)

18. Let fex) be a polynomial function of second degree. If f(l) = f(-I)and a, b, care in AP, then f ' Cal. t ' (b) and t ee) are in :

(a) AI'

(b) GP

(c) HP

(d) arithmetico-geometric progression

19. If Xl' x2' X3 and Yl' Yz, Y3 are both in GP with the same common ratio, then [he points (Xl, Y!), (x2, h) and (X3., Y3):

(a) lie on a straight lint:

(b) lie on an ellipse

(c) lie on a circle

(d) are vertices of a triangle

20. The sum of the radii of inscribed and circumscribed circles for an II sided regular polygon of side a, is :

(a) a cot(~) (b) ~ cot (;)

(c) a cot (!:-) Cd) E. cot (.~-)

271 4 211

21. If in a triangle ABC

a cos2 (%) + e cos2 ( %) = '~J,

then the sides a, band c .

(a) are in AI' (b) arc in GP

(e) are in HP (d) satisfy a -I- b "'c

22. In a triangle ABC, medians AD' and BE are

drawn. If AD. '" 4, L DAB '" ~ and L. ABE '" .j , then the area of the 6 ABC is :

(a) 8/3 (b) 16/3

(e) 32/3 Cd) 64/3

23. The trigonometric equation sin -I x = 2 sin -i a, has a solution for:

(a) 1,.2 <I al < _I_

.. -J2

1 (c)jaj<"2

(b) all real values of a

1 (d)jaj;,;~· ~

J2

24. The upper 3/4th portion of a vertical pole subtends an angle tan -J 3/ Sat a point in the' horizontal

plane through its foot and at a distance 40 ill from the foot. A possible height of the vertical pole is :

(a) 20 m (b) 40 m

(e) 60 m Cd) 80 m

25, The real number x when added to its inverse gives the minimum value ofthe sum at x equals to:

(a) 2 (b) 1

(c) -} (d)-2

26. If f: R ~ R satisfies f( x + Y)= I(x) + fey),

~

for all x, Y ER and f(1) = 7> then E fer) is:

iI"~1

. Tn (a) 2

(c) 7n~n + 1)

27. If f(x) =- x", then the value of f '(1) reI) 1"'(1)

f(l) ~ ~ + -""""21 - """"3! + ...

(-1)" f "(1)

+ is:

Il!

(a) 2" (b) 2"-1

(c) 0 Cd) 1

28. Domain of definition of the function

[(x) '" -~ + 10gJO (xJ - x), is :

4-x

Cal '(1, 2)

(b) (-1, 0) v (1, 2) (c) (1, 2)u (2, m)

(d) (-1,0) v (1,2) v(2, 0))

[ 1 - tan (1)] [1 - sin x]

is:

[ 1 + tan (~) l[n - 2XJ3

29. lim

1 Cal .. c 8

(e) . .!... 32

(b) 0

(d) cc

log (3 + xl-log (3 - x)

30. If lim = k, the value

:<-,0 X

of k is: (a) 0 (e) 2/3

(b) -1/3 (d) -2/3

31. If f n (a), gl1 (a) exist and are not equal for some

n, Further if ita) "" g(a) = k and

lim !(a)g(x) ~ fla),- gea) f(x)+ gea) "" 4

r _,." g(x) - !(x) ,

then the value of k is equal to ;

(a) 4 (b) 2

(c) 1 Cd) 0

32. The function !(:x) = log ex + .~ x2 + 1), is ; (a) an even function

(b) an odd function

(c) a periodic function

(d) neither an even nor an odd function

33. If f(x)={xe~ [I;i ~~] ,x;t: Othen f(x) is:

o ; x=o

(a) continuous as well as differentiable for all x (b) continuous for all x but not differentiable at x=O

(c) neither differentiable nor continuous at x",O

(d) discontinuous everywhere

34.lf the function f(x) = 2x3 - 9axz + 12a2x + 1, where a > 0, attains its maximum and minimum at p and q respectively such that p2 '" q, th~fI. a

equals:

(a) 3 (b) 1

(c) 2 Cd) V2

35. If fey)= el, gey)= y;y > 0

and Fet) '" f~ fCt - y) g(y) dy, then: (a) Fer) = 1 -e-~(1 + t)

(b) FCc)'''' c/ - (1 + t)

(e)F(t)=tl

(d) F(t) = te"

36. If fea + b - x) = fex), then fa" x f(x) dx is equal to:

. a + b JO

(a) -2- a feb - x) dx

(b) a; b J: f(x)dx

b - a fO

(e) --y a f(x) dx

a + b JO

(d) ~ a f(a+ b+ x) dx

2

JX see2t dt

37. The value of lim 0 is :

r ->0 X sin x

(a) 3 (e) 1

(bJ 2 Cd) -1

38. The val ue of the in tegral I '" f ~ x(1 - x)" dx is :

1 (a) n + 1

1 1

(e) 11 'I' 1 - ,,+ 2

(b) _1_. n+2

(d) _. _1_ + _. _1_. n+l n+2

1 + 23 + 33 + .. + 113,

- lim - ---, ---IS:

nc-s cc n.JI

(a) lJ30 (b) 0

(e) 1/4 Cd) 1/5

40. Let :x rex) '" (es: X), x > O.

J4 3 ' 3

If _e"nr dx=F(k)-F(l),

1 X

then one of the possible values of k, is :

(a) 15 (b) 16

(c) 63 (d) 64

41. The area of the region bounded by the curves y = 1 x -]1 and y =,3 -I x] is :

(a) 2 sq unit (b) 3 sq unit

(c) 4 sq unit (d) 6 sq unit

42, Let f(x) be a function satisfying f '(x) '" f(x) with f(O) '" 1 and g(x} be a function that satisfies jex) + gex) = x '. Then the value of the

integral II: f(x) g(x) dx, is :

e2 5 e2 3

(a) e - 2 - ~ (b) e.+ 2' - 2.

~ 3 ; 5

ec) e - -2 ~ 2: (d) e + 2' + 2:

43, The degree and order of the differential equation of the family of all parabolas whose axis is x-axis, are respectively:

(a) 2, 1 (b) 1, 2

(c) 3,2 (d) 2,3

44. The solution of the differential equation

, -1 dy

(l+y2)+,(x~etan Y) dx =O,is:

-1 (a) ex - 2) = ke" Ian y

(b) 2 X et•n -I y '" e2 tan .. 1 y + /( (c) Xe,"n-1Y '" ran " y + k

(d) xe2 [an-1 Y = emn-! y + k

45. If the equation of the locus of a point equidistant from the points (aI' bJ land (a2, b2}is Cal - az) x + (hi - bz) Y + c = 0, then the value of'C'is:

1 2 2 22

(a) 2 Cal + bz - eli - hI)

2 2 2 2

(b) Q1 .- Q2 + hJ - b2

1 2 2 b2 2

(c) '2 Cal + a2 + I + b2)

Cd) Jat + ~2 -ai -bi

46. Locus of centroid of the triangle whose vertices are (acost,asint), (bsint,-bcost) and (1, 0), where t is a parameter, is :

(a) (3x _1)2 + (3yi == a2 - b2

(b) (3x -1/ + (3y)2 = a2 + b2

(c) (:3x + Ii + (3yi == a2 + b2

Cd) (3x+ Ii + (3yi == a2 - b2

47. (f the pair of straight lines Xl - 2p>.y - y2 = 0 and x2 - 2q>.y - y2 = 0 be such that each pair bisects the angle between the other pair, then:

(a) s=« (b)"p=- q

(c) pq == 1 -Cd) pq = - 1

48. A square of side a lies above the x-axis and has one vertex at the origin. The side passing through the origin makes an angle

a (0 < U < *) with _ the positive direction of

x-axis. The equation of its diagonal nor passing through the origin is :

(a) y (cos a -sin a)- x (sin a -cos a)=a (b) y (cos u + sin: 0:) + x (Sin a - cos 0.) '" a (c) y (cos a + sin a) + x (sin 0. + cos a) "" a (d) y (cos 0: + sin aJ+ x (cos a --sin o:)=a

49. If the two circles (x - Ii + (y - 3i = r2 and x2 + y2 - 8x + 2y + 8 == 0 intersect in two distinct points, then : (a) 2 <,. < 8

ec) ree2

(b) r < 2 (d) r » 2

50. The lines 2X - 3y == 5 and 3x - 4y = 7 are diameters of a circle having area as 154 sq unit. Then the equation of the circle is :

(a) x2 + y2 + 2x - 2y == 62

(b) x2 + y2 + 2x - 2y == 47

ec) x2 + y2 - 2x + 2y = 47

(d) x2 + y2 - 2x + 2y = 62

51. The normal at the point (btl, 2 btl) on a parabola meets the parabola again in the point (bd, 2bt 2)' then;

x2 y2

52. The foci of "the ellipse 16 + Il == 1 and the

2 2

hyperbola 1~4 - ~ 1 == ~5 coincide. Then the

value of b2 is :

(a) 1 Ce) 7

(b) 5 Cd) 9

53. A tetrahedron has vertices atO(O,"O,O), A(1, 2, 1), B (2, 1, 3) and C (-1, 1, 2). Then the angle between the faces OAB and ABCwill be :

(a) cos-1 G~J (b) cos-1 GD

ec) 300 (d) 900

54. The radius of the circle in which the sphere xl + y2 + z2 + 2x - 2y - 4z - 19 := 0 is cut by

the plane x + 2y + 2 z + 7 == 0 is ;

(a) 1 (b) 2 .

ee) 3 (d) 4

x-2 y-3 2-4

55. The lines -1- == -1- '" ~ and

x-I y-4 z-5

k- == -2- == -1- are coplanar if:

(a) k == 0 or -1 (b) b= 1 or-l

(e)k=Oor-3 (d)k=3or-3

56. The two lines x = ay + b, Z == cy + d and x = a' y + b', z == c' y + d' will be perpendicular, if and only if :

(a) aa' +bb' + cc' + 1 == 0 (b) acr' + bb' + cc' = 0

(e) (a.+ cr')(b + b')-+ (e+ c')= 0 (d) aa' + cc' + 1 = 0

57. The shortest distance from the

12x + 4y + 3z == 327 to the

xl + y2 + Z2 + 4x - 2y - 62:= 155 is :

4

(a) 26 . (b) 11 13

(e) 13 (d) 39

plane sphere

~8. Two systems-of rectangular axes have the same origin. If a .plane cuts. them at distances a, b, c and a' , b' , c' from the origin, then:

1:, 1 1 1 1 1

(a) - + -+ -+ -+ -+ -= 0

c? b2 c2 a,2 b'z e''2.

- 1 1 1· 1 1 1

(b) -+ --,- +-+ ---=0

a2 b2 c'- a,2 b,2 C,2

1 1 1 1 1 1

ee) - - - - - + - - - - - == 0

02 b~ c2 a,2 b,2 C,2

1.-i 1 111

(d) - + - + - - - - - - - == 0

a2 b2 c2 a,2· b,2 C,2

~ -.7 4-

59. a, D, C are three vectors, such

4- -.7 ~ 7 4- ~. -» a + D + C = U, I a 1 = 1, 1 b 1 "" 2. 1 C

-,)~ ~-» ~-».

then a. b + b- C + c- a IS equal to :

dlat

1= 3,

(a) 0 (c) 7

(b) -7 Cd) 1

-,) ---0> 4-

60. If u, v and ware three non-coplanar vectors,

~4~ 44--Jo~

then; u + v -w)· [(u - v) x( v -w)1 equals:

(a) 0

(b) tr·~ x ~

(c)ll~x1 (d)31t.~ x~

61. Consider points A, B, C and D with position

vectors 7 i - 4 j -I- 7 k. i - 6 J + 10 k.

-1-31+41.. and sl-1+sk respectively. Then ABCD is a :

(a) square

(b) rhombus

ec) rectangle

(d) parallelogram bur not a rhombus

62. The

and

vectors

~ ...... "... .......

AC = 5 i - 2 j + 4 k are- the sides of a triangle

ABC_ The length of the median through A is:

(a) .Jls (b) m

(c) 53 (d) ,[288

63. A particle acted -on by constant forces

4 i + 1 - 3 k and 3 i + 1 - k is displaced from the point! + 21 + 3k to the point si + 41 + k. The total work done by the forces is :

(a) 20 unit (0) 30 unit

(c) 40 unit (d) 50 unit

---+ ......... --)."...,'" -----) ............ ""

64. Let u = i + J. v ~ i - j and w '" i + 2 j + 3 k.

. If ~ is a unit vector such that \t. ~ =: 0 and

---+ ~ - ---+ -

v • n = 0, then 1 w- 11 ! is equal to :

(a) 0 (b) 1

(c) 2 Cd) 3

65. The median of a set of 9 distinct observations is 20.5. If each of the largest 4 observations of the set is increased by 2, then the median of the new set:

(a) is increased by 2 (b) is decreased by 2

(c) is two times the anginal median

(d) remains the same as that of the original set

66. ln an experiment with 15 observations on x, the following results were available

E Xl = 2830, L x = 170 .

One observation that was 20, was found to be. wrong and was replaced by the correct value 30. Then the corrected variance is :

(a) 78.00 (b) 188.66

(c) 177.33 Cd) 8.33

67. Five horses are in a race. Mr. A selects two of the horses at random and bets on them, The probability that Mr. A selected the winning horse, is :

(a) i

5

(c) ~. 5

(b) ~ 5

Cd) ~ 5

68. Events A, B, C arc mutually exclusive events

3x + 1 1- x

such that peA) '" -3-' PCB) '" -4- and

PCC) = 1 -22x . The set of possible values of x

are in the interval:

(a) [~, ~J

(e) [~, liJ

(b{1' ~]

(d)[O,1]

69. The mean and variance of a random variable X having a binomial distribution arc 4 and 2 respectively. then P(X '" 1) is :

1 1

(a) 32 (b) 16

1 1

(c) "8 Cd) 4

-Jo ~ --+ ---7

70. The resultant of forces P and Q is R. If Q is

~

doubled. then R is doubled. If the direction of

~ ---+

Q is reversed, then R is again doubled, then

p2 ;Q2: R2 is:

(a)3:1:1 (c) 1: 2: 3

(b)2:3:2 (d)2:3:1

71. Let Rl and R2 respectively be the maximum ranges up and down an inclined plane and R be the maximum range on the horizontal plane. Then R1, R, R 2 are in :

(a) arithrnetico-geometric progression (AGP) (b) AP

ec) GP

Cd) HP



72. A couple is of moment G and the force forming

---+ ~

the couple is P. If P is turned through a right

angle. the moment of the couple thus formed is

~ 4-

H. If instead, the forces P is turned through an

angle u, then the moment of couple becomes :

-+ -~ 74. Two stones are projected from the top of a cliff
(a) G sin IX - H cos Ct h. metres high, with the same speed u so as to hit
...:; -+ [he ground at the same spot. I f one of the stones
(b) H cos C( + G sin (t is projected horizontally and the other is
-) -+ projected at an angle e to the horizontal, then
C c) G cos IX + H sin Ct tan e equals :
-) -+ f?; (b) 2g f~
(d) H sin (t - G cos Ct (a) .
73. Two panicles start simultaneously from the (c) 2hK- Cd) ufj;
same poim and move along two straight lines,
one with uniform velocity ti and the other from 75. A body travels a distance s in t seconds. It starts
...:; from rest.and ends at rest, In the first part ofthe
rest with uniform acceleration f. Let tt be the
angle between their directions of motion. The journey, it moves with constant acceleration f
relative velocity of the second panicle w.r.t. the and in the second part with constant
first is least after a time: retardation r. The value oft is given by:
(a) u sin a (b) f cos a (a) 25 (] +~) (b)~
1 1
f u -- + -
(d)uc9sa f r
(c) L/ sin C1. (d) ~2S (7 +;)
f (c) .j2s(j + r)
/MNSWERS
III. PHYSICS AND CHEMISTRY
1. (b) 2. (a) 3. (b) 4. (a) 5. (d) 6. (a) 7. (a) 8. (d)
9. (e) 10. (b) 1l. (a) 12. ee) 13. (e) 14. (d) 15. (e) 16. (a)
17. (a) 18. (e) 19. (c)' 20. (b) 21. Ce) 22. Cd) 23. (d) 24. . (b)
25. ~d) 26. (a) 27. Cd)' 28. (c) 29. (e) 30. (b) 31. Cd) 32. (a)
33. (d) 34. ee) 35. (e) 36. (a) 37. (c) 38. (d) 39. (a) 40. (d)
4l. (c) 42. (a) 43. (d) 44. (a) 45. (b) 46: (a) 47. (a) 48. (b)
49. (b 50. (c) 51. ee) 52. Cd) 53. (b) 54. (d) 55 . (e) 56. (b)
57. (b) 58. (a) 59. ecl . 60. (a) 61. (d) 62. (b) 63. (e) 64. (a)
65. (a) 66. (b) 67. (a) 68. Cd) 69. (b) 70. (c) 71. (c) 72. (a)
73. (b) 74. (d) 75. (d) 76. (b) 77. (a) 78. eb) 79. (a) 80. (b)
81. (b) 82. (c), 83. Cd) 84. (d) 85. (b) 86. (e) 87. (a) 88. (b)
89. .(d) 90. (b) 91. ec) 92. (c) 93. ee) 94. (b) 95. ecl 96. (b)
97. (b) 98. ee) 99. (b) 100. Ce) 10l. (a) 102. eb) 103. ec) 104. (a)
105. ed) 106. (a) 107. (d) 108. (d) 109. (e) 110. (b) 111. (a) 112. (e)
113. Cd) 114. (b) 115. Cb) 116. Ce) . 111. (a) 118. (c) 119. (a) 120. (b)
121. (b) 12.2. (a) 123. (d) 124. (b) 125. (b) 126. (b) 127. (a) 128. (d)
129. (e) 130. (a) 131. (b) 132. (b) 133. (c) 134. (c) 135. (d) 136. (b)
137. (a) 138. (d) 139. (a) 140. (e) 141. (a) 142. (b) 143. (c) 144. (a)
145. (a) 146. (d) 147. (d) 148. (e) 149. (d) 150. . (e)
",,.. MATHEMATICS
1. (c) 2. (c) 3. (d) 4. (a) 5. (b) 6. ec) 7. (c) 8. (b)
9. (a) 10. eb) 11. (b) 12. (a) 13. (a) 14. (b) 15. (b) 16. (e)
17. (d) 18. (a) 19. (a) 20. (b) 21. (a) 22. ("') .23. (a) 24. (b)
25. (c) 26. (d) 27. (el 28. (d) 29. ee) 30. Cc) 31. (a) 32. (b)
33. (b) 34. (e) 35. (b) 36. (b) 37. (c) 38. (e) 39. Cd) 40. (d)
41. (c) 42. ee) 43. (b) 44. (b) 45. (a) 46. (b) 47. (d) 48. (d)
49. (a) 50. (e) 51. (a) 52. (c) 53. (a) 54. (c) 55. (e) 56 .. (d)
57. (c) 58. (d) 59. '(b) 60. (b) 61. C") 62. (e) 63. (e) 64. (d)
65. Cd) 66. (a) . 67. (d) 68. (a) 69. (a) 70. (b) 71. (d) 72. (c)
73. (d) 74. (d) 75. (d)
(x) No option is correct in question paper. HlNTS fI SOLUTIONS

1. When particle describes circular path in a magnetic field, its velocity is always perpendicular to the magnetic force.

~-4

Power P = F . v 0: Fv cos 9

Here, 0= 90°

But

P",O W

P=t

W =p·t

Hence, work done W=O

2. The force on a particle is y

. (everywhere)

z

So,

~ -4-+-+

F=q(E+vxB)

or

-+ -+ -+ F=Fe+Fm

:.1. = q"1 =-16 X 10-18 xl04 C-k) = 16 x 1O-1~ k

-+ J8 ~ ~

and F", '" - 16 x 10- (10 i x B j)

=,-16xl0-17 xB(+k) =-16xl0-17 Bli:

Since, particle wi!! continue to move along + x-axis, so resultant force is equal to O.

-4 ..,+

F.+F",=O

16 x 10-14 =16xlO-17 B

B = 16·:x 10-14 = 103 16 xl0-17

B = 103 .wb/m2

3. When magnet -is divided into two equal parts, the magnetic dipole moment

I M M' = pole strength x"2 = 2"

[pole strength remains same] Also, the mass of magnet becomes half i. e. ,

, m m=2

Moment of inertia of magnet mil

[= "12



New moment of inertia

I' = 112 (-i-) Gf = 1r~28

[' .: 8

Now, T '" 2n ~(~B-)

TO = 2n ~(~:B) = 2rr (~~~2)

TO=?':' => TO 1

2 Y='2

W = MB (1 - cos 0)

W =MB (1 - cos 60°) W=MB

2

. MB=2W

Torque, T '" MB sin 600 ME.J3 2W.J3

=-2-=-2-

=w)3

5. Inside bar magnet, lines of force are from south to north.

6. At Curie temperature, a ferromagnetic substance transits into paramagnetic substance.

7. In stational)' position, spring balance reading = mg "" 49

In "" 49 = 5 kg 9.8

When lift moves downward. mg - T "" mG

Reading of balance .

T==mg-ma

== 5 (9.8- 5) = 5x 4.8 "'24.0N

mg

8.

Vu:;! V l E L

where, l == balance point

L = length of potentiometer wire I

or V = I E

V = 30 x E = 30. E

100 100

9. Germanium is semiconductor, whereas copper is conductor.

For conductors, R oc A t For semiconductors,

. 1

R oc-

At

Hence, when both are cooled, then resistance of copper decreases whereas that of germanium increases.

10. Some of the characteristics of an optical fibre are as follows :

(i) This works on the principle of total internal reflection.

(ii) It consists of core made up of

glass/silica/plastic with refractive index n1, which is surrounded by a glass or plastic cladding with refractive index nz(nz > nj)' The refractive index of cladding can be either changing abruptly or gradually changing (graded index fibre).

(iii) There is a very little transmission loss through optical fibres.

(iv) There is no interference from stray electric and magnetic fields to the signals through optical fibres.

11. Thermo-emf of thermocouple = 25j.l.V;oC.

Let 8 be the smallest temperature difference. Therefore, after connecting the thermocouple with the galvanometer, thermo-emf

E == (25IlV;oC) xiWC) = 258 x 10-6 V

Potential drop developed across the

galvanometer

== iR = 10,,5 X 40 = 4 X 10-4 V

4x10~4 '" 258x10-6'

12. According to Faraday's law of electrolysis, III =: zit

m~.!l. = zzn.

IIlcu zCu

0.13 32.5 mCn = 31.5

mcu =; 0.126 g

13. As we know that formula of velocity is 1

y=--_

)~Of'O

'y2 = _1_ == [LT ·-1 ]2 Il oEo

_1_=[eT-2]

Il OlOO .

14. Mass of disc (X), nlx == nR2tp

where, p '" density of material of disc

1 . 2 1 2 2

IX"'2.mxR =2.nR [pR

1 4 Ix == 2. 1lptR

Mass of disc (Y)

my'" 1l (4Ri % p == 4nR2tp

1 2 1 2 6 2

t, = 2. my (4R) == 2. 4nR tp·1 R

Iy == 327ttpR4 ... (ii)

... (i)

and

Iy 321ltpR4 T:" 1

x -1lptR4

2

= 64

.. Iy=64Ix

15. According to Kepler's law T 2 oc 1'3

52 a; 1'3

(f 'i cc(4rl prom Eqs. (i) and (ii)

25 1'3

(f'i = 641'3 T '= .J1600 '['=40h

... 0) ... (ii)

or

16. Angular momentum

L == [(f) ... (i)

Kinetic energy

K == ~ 10/ = ~ L(O [from Eq. (i)]

L= 2K

(j)

L'= 2(~1

20)

L' == !:_

4

Now;

18.

~ \V--)o

[Before decay]

v<---® @--)oU [After DecayJ Apply conservation of linear momentum. 0= 4u - 234v

4u

=> y = 234

The residual nucleus will recoil with a velocity

f 4u .

0234 unit.

Recoil speed of residual nucleus is 2~~'

Note : If they will ask the recoil velocity, then

. ,4u dt4U

answer remains same t.e., 234 an no - 234 as

the word recoil itself is signifying the direction of motion of residual nucleus.

19. Let at '0' there will be a collision. If smaller sphere moves x distance to reach at 0, then bigger sphere will move a distance of (9R - x).

5M

F"" GMxSM (12R - xi.

F G xSM

asrnolJ == M = (12R _ X)2

F GM

~ig == SM = (12R _ X)2

i 2' ==.!. G x SM t2 0

x == '2 a..~l"lIt 2 (i2R _ xi ... I

(9 R - x) ==.!.2 ~ig t2.!. GM 2 t2 ••• (ii)

2 (12R - x)

Thus, dividlngEq. (i) by Eq. (ii), we get

.. __ x_=s

9R -x

::::> x e= 4SR - 5x

=> 6x =: 45R

=> x=7.SR

20. The difference in the variation of resistance with temperature in metal and semiconductor is caused due to difference in the variation of the number of charge carriers with temperature.

21. v2 = u2 + :?as

0== (50 x l~r + 2a x 6

a =-16 m/s2

(a == retardation)

Again y2 ",u2 + 2as

(. 5)2 .

0"" ,100)(18 -16x2)(5

(100 x 5)2

S == 18 x18,x 32 == 24.1",24 ill

22. The ball will be at point P when it is at a height of 10 ill from the ground. So, we have to find the distance OP, wh ich can be calculated directly by considering it as a projectile on a levelled plane (OX).

torn/s

Ground

OP == R = ~2 sin 2.Q == 102 x sin (2 x300)

g 10

= lO-J3 == 5J3 2

= 8.66 m

23. To increase the range of ammeter we have to connect a small resistance in ___2_ A..L1iih '~ parallel (shunt), let its value ~

be R. 0.810

Apply KCL at junction to divide the current.

R

~

,~

lOA 1A 0.810,

Voltage across R = Voltage across ammeter.

~ 9R =0.81 xl

, => R == 0.:1 = 0.090

24. Planck's constant (11) == J - s[MI?r2] [T) [MLlrl]

Momentum (p) kg - m - 5-1

= [M][L][rl] ",(MLT-I]

25. Resultant force is zero, as three forces acting on the particle can be represented in magnitude . and direction by. three sides of a triangle in same order. Hence, by Newton's 2nd law

( ._} l

! 1 = 171 dd v ,particle velocity ("1) will be

I, t )

same.

26 [F' G·. I Charge enclosed

.. rom ,auss aw, --"----

. '1l

= Flux leaving the surface]

_g_ = <h - ~1

£0

q "" ($2 - ~) tio

27, Let R he the normal contact force by wall on the block.

R =ION 10N

Ii. = IV and f 70 ~R

~tR ~'w

= w = O.~ x 10 = 2 N

28. LeI coefficient of friction

be p, then retardation will be ~lg.

From equation of motion, v = u~· at

='> 0 ~ 6 - ~lg x 10

6

=- f1 = 100 = 0.06

29. Here B is implying A bur A is not implying B, as kinetic energy of a sysrem of particles is zero means speed of each and every particle is zero which sa ys tha t rnorne nturn of every particle is zero.

But statement A means linear momentum of a system of particles is zero, which may be true even if particles have equal and opposite momentums and hence, having non-zero 1m.

30. Mutual inductance of the pair of coils depends on distance between two coils and geometry of two coils.

31. Let acceleration of system (rope + block) is Q along the direction of applied force. Then

p

a= M + m

w

--:.a

M

Draw the FBD of block and rope as shown in figure.

_8

_8

m

T --fJlJJj!h!i(:i;'l::~:!Jf'Jf.,/jI(~'.'iii{!f'.:'iii'.1· • p

Where, T is the required parameter

For block T = Ma

MP T=--M+m

32. The arrangement is shown in figure.

Light s pri n 9 balance

M

Now, draw the free body diagram of the spring balances and block.

~1

BlOCky

Mg

Tl

For equilibrium of block, T] '" Mg where, 1'1 = Reading of 52

For equilibrium of 82, T2 "" 11 where, T 2 = Reading of 51

For equilibrium of St, T2 = T3

Hence. TI = T2 '" Mg

So, both scales read M kg.

33. Elastic energy stored in the wire is

U 1 . 1

= 2 stress x strain x vo urnc

1 F t.l

='2'1fxT"xAL

1

='2F Al

'" ~ x 200 x 1 X 10.-3"" 0.1 J

34. The escape velocity is independent of anglc of projection, hence, it will remain same i. e., 11'

km/s, .

35.

... (ii)

•.. (i)

Dividing Eq. 0) by CU), we have

.. ~~JM~ m

9 M 25 M+ m

=> 9M + 9m "" 25M

=> 16M =9m

m 16 -M=9

36. Heat cannot flow itself from a lower temperature to a body of higher temperature. This corresponds to' second law of thermodynamics.

37.

(given)

Hence,

38; T '" 21!-f[

Given,

AT 1 Al T='2T

jl"'21%

AT 1

T = '2 x21% '" 10.5%

39. The given equation of wave

y = 10-4 sin (600t - 2x ... -j) .. ,.(1)

Standard equation of wave

y",aSin (rot =kx + $) .. \(ii)

Now comparing Eqs, el) and (ii), we get,

(t) '" 600 and k ~ 2

. _ ro 600

velocity of wave", k '" 2 = 300 m/ s

40.e=_Ldi=_L (-2-2)

dt 0.05

(4)

8 =L 0.05

L '" 8 x ~.05 == 0.1 H

41. In an LC circuit the energy oscillates between inductor (i~ the magnetic field) and capacitor (in the electric field).

U t: max [Maximum energy stored in capacitor] Q2

2C

U B max [Maximum energy stored in ind uctor] Le

=__:Q_

2

where, To is the current at this time. For the given instant U!! '" U B

q2 LiZ 2C=Z

i .. f.,

From energy conservation

VE + VB = V E max'" U B max

=>

q2 Q2

2 2C' ~2C

q= Jz

42. The core of transformer is laminated to reduce energy loss due to eddy currents.

--)0 4- ---'t

43. .. t=orl<F

---'t ---'t ---'t ---'t---'t

t is perpendicular to both rand F, so r· t as

---'t---'t

well as F - t has to be zero.

44. Given: No Ie = 5000, N Ie = 1250 N = No e~x{ = No e-s).

Where '}~ is decay constant per min N l=No Ice--s}.

1250·= Nn A e-·s).

NoA = 5000 = 4 No le-Si• 1250

(5)" =0 4

5" = z log, 2 k= OAin 2

45. Since, 8 «-pardcles 40- -parucles and 213'particles are emitted, so new atomic number

Z'=Z~8x2+4)<1-2xl = 92-16 + 4 - 2

'" 92-14

=78

, 1 2

46. hf '" hfo + 2: mv

2 2hfl 2hfo

Hence, vI = -m - -TTl

2 2 2h

VI - V2 = - [fl - f2) m

47. Protons cannot be emitted by radioactive substances during their decay.

48. Resistance in the arms AC and BC are in series, A

3V

3(2 R'=3+3",6n Now R' and 3 n are in parallel,

R ",6x3=2"

.' 'q 6 + 3 H

Now, V=IR

3

I", 2'" 1.5 A

49. Aluminium is a metal, so when we insert an aluminium foil, equal and opposite charges appear on its two surfaces. Since, it is of negligible thickness, it will not. effect the capacitance.

Alternative : From the formula

C '" t::oA

d-t+.E__ K

=:> Here, K '" C1J and t ---)0 a

eoA EoA

So, C 0; ~-:O = --or ""Co

50. x = 4 (cos nt + sin ztr )

= ~ x../2 [cos ltt + sin TIt]

,..'2 .

x '" ill sin [ TIt + % ]

So , amplitude = 4.fi

51. At P due to shell, potential q

Vi ~-~- 4TI Co R

at P due to Q, potential

"

--d-

Q

V2 =---If

411 Co2

2Q 41l£oR

:. Net potential at P V"'\'J+V2

P UR/2 R Q

=_q-+~

4TIEoR 4TIEoR

2

52. W '" ~ CV2 '" ~ 2

1 (8 x 1O-18i

=- x ---

2 100 X 10-6

'" _!_ x 64 X 10-36 = 32 X 10-32 J 2 100 x 10-6

53. x = at 3, Y =~t 3

dx 2

V ",-=3a.t

-x: dt

v, = i '" 313t2 Resultant velocity

_---

/ 2 2

v=cv, + Vy

=~9a2t4 + 9132t4 =3t2~a2+~2

54. Given: P «T 3 ... (i)

In an adiabatic process

T Y pI -, y '" constant 1

T oc----

p(l-y)/y

T (yly-I) ocp

q

... (ii)

Comparing Eqs. (i) and (ii), We get

., _Y-=3

)'-1

31' - 3=y 2y= 3 c, 3 ~=Y=2:

55. Work does not characterise the thermodynamic state of matter, it is a path function giving only relationship between two quantities,

56. TJ '= 627 + 273 = 900 K

Q1 = 3x 106 cal

T2 = 27 + 273 = 300 K QJ Q2

., T; == '[2

57.

T2 300. 6

Q2 '" lj x QJ = 900 x 3 x 10

'" '1 x 106 cal Work done =Q1 - Ql

= 3 x 10 - Ix 106"" 2 X 106 cal :. 2 x 4.2 x 10C, J= S.4x 106 J

1 2 1 3 '22

WJ = 2 k x Xl :;:2 x 5 x 10 x (5 x 10-)

= 6.25 J

1 2

.W. ~, 2. k (XI + x::J

= ~ x 5 x 103 (5 x 10-2 + 5 x 10-2)2 <= 25 J

Net work done = Wz - WJ

'" 25- 6.25= 18.75 J

'" l8.75N-m

5&. The wire will vibrate with the same frequency as that of source. This can be considered as an example of forced vibration.

T = 10x9.8N'='98N, m = 9.8 x 10-3 kg/m

. 1 ~('r)'-

Frequency of wire f '" .;-_ --

2t m

1 19S-)

=0 2 x 1 'II 9.8 x 10-J

"" 50 Hz.

59. I,= 256 Hz

For tuning fork f" - Ii = ± 5, f2 <= frequency of piano

12 <= (256 + 5) Hz, or (256 - 5) Hz

When tension is increased, the beat frequency decreases to 2 beats/so

If we assume tha I the frequency of piano string is 261Hz, then on increasing tension, frequency, more than 261Hz. But it is given that beat frequency decreases to 2, therefore 26] is not possible.

Hence, 251 Hz i.e., 256-5 was the frequency of piano string before increasing rension.

60. At X= 0, kinetic energy is maximum and

, potential energy is minimum.

61. ~ kT == 7.7 x 10-14 J

T _ 2x 77x 1O,·H - 3 x 1.38 x 10-23 0: 3.7x 109 K

62. A;; ssCSI3:\ has larger size among the four atoms given, thus, electrons present in the outermost orbit will be away from the nucleus and the electrostatic force experienced by electrons due

to nucleus will be mnurnum, Therefore, the energy required to liberate electrons from outer orbit will be minimum in case of ssCS1J3.

64. Due to the reverse biasing the width of depletion region increases and current flowing through the diode is almost zero. And in this case electric field is almost zero at the middle of the depletion region.

213.6

65. E ~ - Z -;:;2 eV

For first excited state

E., = - 32 x 13.6

f. 4-

<=- 30.6 eV

Ionisation energy for first excited state of Li21 is 30.6 eV.

66. P <= constant

[lv",P

=>

t·: P = force x velocity] [": F '" Mal

Ma x v = P P VQ"'Jj

V x [::_~.] =!...

ds M

Iv., I ,I' P

v" dv = - ds

o uM

[Assuming at t = 0 it starts from rest, i.e., from s '" OJ

=>

, =>

=>

=> S <LC1n

67. Here, thrust force is responsible to accelerate

the rocket, so initial thrust of the blast = rna

'" 3.5 X 1.04 xlO = 3.5 x lOs J\i"

.68. Sustained interference is possible with coherent sources only.

69. Force on - ql

---j. 1 qq' 1 qq , , F == -- ___l__1_ i + -- ___l,-l [sin Oi - cos OJl

4:rrf{) b2 4Il£o a2 .

From above, x I component - q 1 F 12

of force is ~.

F == ___!h_ l q2+ q3 sin 0] . e

x 47t8o b2 a2 :

l l: F

F· q2 % in OJ: 13

.. ix: - + - sm' .

Xb2 02

v2 2

70. R =_ '" (220) P 1000

where, V and P are denoting rated voltage and power respectively

P = Va2pPlied = 110 x 110 x 1000

. . consumed R 220 x 220

=2S0W

71. Objective of compound microscope is a convex lens. Convex lens forms real and enlarged image when an object is placed between its focus and lens.

7

76. Hydrogen spectrum coloured radiation means visible radiation corresponds to Balmer series (nl == 2, 112 '" 3,4 ..... )

380 nm 780 nm visible

V!BGYOR I

CH --)- 2nd orbit

-----?

-------:)-

3rd line from the red end it means 5 -+ 2

77. ;., == _!!_ == 1.105 X 10-33 m mv

78. For s-electron, I == O.

3600 n=O--l

3= 3600 -1 I)

0= 90°

74. According to Newton's law of cooling

dQ ocA8

dt

But dQ a; (A Or (given) dt

73.

.. n = 1

75. Given: I' = 1+ 100% 1 = 21

Initial volume = final volume nr21 = 1tr,2 I'

,2 r21 2 I

r =r=' X2i

i.e.

r2 r,2=_

2

r 2/

R'==PA,==P-----;2 1(r

p·41

1t~

=4R

Thus,.. !ill = R' - R = 4R - R

=3R 3R

. % AR =.- x 100% = 300%

.. R,

79. Mass of one unit-cell (m) == volume x density == oj xd

3 Mz Mz

"'a x--· -=-.

No a3 No

58.5 x 4

m= g

6.02 x 1023

.. Number of unit cells in 1 g == _!_

m

6.02)( 1023 ._ 2.57 X 1021 58.5>( 4

80. Glass is a supercooled liquid, which forms a non-crystalline solid. The common lime-soda glass used for bottles and jars is a supercooled mixture of sodium and calcium silicates.

8·1. NH3·an inorganic solvent.

AgI is having much higher covalent characteristics iri comparison to AgCI.

82. AU physical and chemical properties of elements ate periodic function of atomic number- Modem periodic law.

83. C-atorns form covalently bonded plates (layers). Layers are bonded weakly together, that's Why one layer can slfde over other cause lubricacy. Cannot be melted easily as large number of atoms being bonded strongly in [he layer form big entity.

o

II

84. CH3 -C -CH- CH3

I 2 13 4

CH3 3-mcrhyl. butan-z-one

or 3"methyI2·bul'i\n(}[\e

o II

keto (-~C --:- ) functional group is given priority,

85. LiA!K4 reduces -CDDH to ~CHPH without affecting C = C bond.

86. In between two successive collision, no force is acting on the gas molecules, Resultantly it travels with uniform velocity during this interval, and hence, it moves along a straight line.

87. Cu, Ag, Au group of elements arc called coinage metals as these are used in minting coins.

88. C"HZ,,02 is general formula for open chain add and ester

Il == 3 C~H602

Acid

-0

II

CHsCHz -C-O-H

o 1\

Bster eHa -C-O-CH3

89. Alcohol has polar Ii which makes inter-molecular H-bonding possible. Bther is non polar hence no Hsbending. Lack of H -bonding in ether makes it more volatile than alcohol. .

90. Chiral compounds which have one chira! centre, All four atoms or groups attached to carbon are different

91. Alkaline earth metal salts are causing hardness:

Temporary hardness caused by soluble Ca and Mg hydrogen carbonates .. calcium and magnesium soluble sulphates and chlorides cause permanent hardness.

92. Molec~le Hybri.di~ Repulsion sation

S02 sp2 Ip - bp. bp - bp

Bond angle

119"

104.5°

OH2 sp' Ip - Ip, bp -Ip
bp-bp
SA:! Sp3 -do-
NH sp3 Ip - nfl, bp - bp
~ 90"

lOr

98. N02 and 03 both are having irregular geometry.

94. N3~ 7 + 3= 10 electrons

F~ 9 + 1 '" 10 electrons Na"" 11 -1 '" 10 electrons

Resonating (canonical) structu res of formate ion

96. In CO 2' C·atom is.sp-hybridised thus it has linear structure.

ln XeFz, Xe is sp3d hybridised with three lone-pairs of electrons on equatorial. position. This minimises repulsion hence it has also linear

structure.

Cr: [Ar] I tit J tit I tI f I

3d5 4S'

c-. [Ar) I tl t I tit It I

3d5 4sQ

_97.

(by first IP)

This is stable EC hence romlatiol' of Cr'" by second IP requites maximum enthalpy.

98.

6HQ", - \Ie

Takes place with decrease in number of mole or pressure hence lncrcase in pressure shifls eqnillbrium in COIWrlrd side.

Tokes place with evolution of bent or increase in remperaru rc lienee decrease in IC m peram re sh i fts I his equilibriumin forward

, side.

99. 2BC13+ 3H2 ----+ 2B + 6HCI

2 I 3 I 2m,,1 .

mo mo 21._6 g ~ 2 mol

21.6 g B '" 2 mol B'" 3 mol H2 PV = nRT

. V'" nRT =3x 0..0.821 x 273 672 L

.. P 1 -.

[NO 12 (1.2 x 1O-2P

100. K '" _~2 _

- c [NP 41 4.8 x 10-2

'" 3 X 10.-3 mol L-1 ior. AB2 ~ A2+ + 2 B-

- S 25

Ksp '" [A2" J [B-J2

== (5) C2SP '" 453 =4(lx1C-5i

'" 4 x1C"15

102. Ag+ + e- ~ Ag

965CC == C.IF'" 0..1 equivalent Ag = 0..1 mol Ag

'" 10.8 gAg

- EO 0.0.591 I Q

103. Eeen == <ell - --. ~ og rt ,

Cu 2+ + Zn -------7 Zn 2+ + Cu

~lM 1M

Q = [Z02+] =-~= 10. [Cu 2+] 0..1

E '" 1.10.- 0..0.591 log 10.

cell Z

= 1.10.- 0..0.295 = 1.0.705 V

104. t:-.T f '" molality x K J xi =o..2x1.85)(13

HX - -- H+ +X-1 Ct == 0..3

'" 0..4810 i = 1 + Ct '" 1.3

:.fp "'-0.48IQ~

105. Rate becomes xY times lfconcentration is made x times of a reactant giving /h order reaction.

Rate = k [At [Blm

Concentration of A is doubled hence x '" 2, y = n and rate becomes == Zn times

Concentration of B is halved, hence x '" 1/2 and

(l)m

y == m and rate becomes = 2. times

(l)m _

Net rate becomes = (2)n 2, times

'" {2t- m times

106. Let molarity ofBa(OH)2 '" M 1

. . Normality = 2 M I

Molarity ofHCI = 0.1 M = 0.1 N

NJ\.'t'=N2V2 2M 1 x 25 == 0.1 x 35 MJ =O.07M

107. t:.G'" "" - Wm~". Maximum work done by the system.

108.CHz=CH2 + H2 -------7 CH3 -CH3 AH "" (BE),.;ttanl' - (BE)",oduns

== 4 (BE)c_H + (BE)c=c + (BE)H_H ~[6(BE)c_H + {BE)c_d

=-I25kJ

109. Enthalpy change is state function and depends only on initial and final condition do not depend on path or interrnediares.

110. High pressure inside the cooker increases boiling point, thus heat consistently given is used up by food material instead in boiling and constant boiling evaporation.

Ill. Ideal solution b. H = 0 t:-.v= 0. [;A-A == FH -s= FA_B

112. (~n = k [NO]2 (02)

"'k(~~Or(n~z) (:) '" ~ (rtNoiCn02)

(:) ~ k ("(~;no,)

:8(~;)

113.

EO _ 2.303RT I . K

((!!1 - nF og eq

D. 295 = 0.0591 I K

. 2 og eq

logKeq = 10

K "" 1010

oq

114. An irreversible process => spontaneous process

= (dS)\IE (change in entropy) == + ve > 0 - = (dG)y. p (change in Gibbs free-energy)

-ve=><O

115. As temperature increases desorption increases.

Adsorbent + Adsorbate ~ Adsorbed state + 6E Adsorption is exothermic process (forward direction), desorption is endothermic process (Backward direction)

According to Le-Chatelier's principleincrease in temperature favours endothermic process,

116. k '" A e-E~/IIT

k ea Rate constant

A '" Pre-exponential, frequency factor EQ '" Activation energy

R = Gas constant

T = Temperature

117. More ~ vc value of EQ means larger reducing power.

H8. Proton affinity it means affinity for proton l.e., basiciry

NHrNitrogen has pair of electron to donate as well as higher tendency to donate due to lower electronegativiry. P is not suitable as that has larger size.

119. ZnO can react with acid and base both.

ZnO + 2HCl -----)0 ZnC12 + HP ZnO + 2NaOH ...._.. NazZnO 2 + HP 120. HgI2 + 2K 1 -------+ K2Hgl4

II . soluble

Hglz -------+ Hg + 12

121. ~HCl + -102 ~ Hp + Cl2

122. In Holmes signals of the ship mixture of CaC2 andCaJPz is used.

123. Fe26• [Ar] 3d6 4s2

Fe2t (24elccmms) ~[Ar] 3d6 4so

124. 2CrO~- + 2H+ ...._.. Cr20i- + Hp 125 .. K4[Ni(CN)4] --+ 4K+ + (Ni{CN)414- X + ( 4 x ~ 1) '" ~ 4 x~4=~4 x=o

126. The pair of electron present with nitrogen will not be available to be donated as W will consume that one.

127. CO(NH3)sci] Cl2 __,_ [Co(NH3 )5CI]2+ + 2CC

Ihree ions

2CI- + 2Ag+ ~ 2AgCI

free

2 mol 2 mol 2 mol

128. Due to lanthanide contraction there occurs net decrease in size. Only one 0.85 'A is smaller one.

-p -p ··U

129. ~~4Th ~ ~ ~tjx ---} ~~oy

130.C =Co Gr

Total lime 18 6

y =--,--_. =- '"

TIn 3

Co'" 256g

C ~ 256 (l)6 '" 256 4

(uudcccycdl ~ . - 2 64 =;. U

131. Factual

1 . 1 S""in·'

132. CaSO 4' 2: H.p + 12: H20 ~

Hardening

CaSO 4 . 2H20 CaSO 1 . 2H20

Gypsum Gypsum

(orthorhombtc) (monoclinic)

133.1 i< 10-8 M Hel solution Hp i5al50 present there which also undergoes self ionisation HP'~H+ +OH-

10,,7 M at 25"C

If it is taken simply even without co, I ion effect, higher concentration must be conx.-Iered which is 10-7 M but H<' from HC] decreases self ionization which further decrease self ionization, hence, [H'] from Hp.

Hence, net concentration must be smaller than lO-"1M

Hence, acidic

134. NH3, CH3NH2, (CH3)2NH

K, ..

(CH3 hNH + Hp .,..- eel'I:l)2 NI f2 + OH-

r effect maximum stabilization more Hsbond stability high, hence, most basic

K2 4

CHJNH2 + HP ...---" CH3NH3 + 01-1- K3·

NH~ + Hp ..----" NH; + OH-

KJ >K~ > K~

Hence, basicity order is

(CH3)2NH >CH3NH2 > NH3 PdlH2 135.CH2=CH-CHzCH3 ~

Butelle-l ii, pres:sllr~~

CH3 -C!i2 -CHz -CHJ

hUU~!lC

other reagents arc successful with polar double bonds.

·136. Solubilities of carbonates decreases down the group because latticeenergy decrease is almost constant while decrease in hydration energy happens sharply .. Finally difference of hydration energy and lattice energy decreases thus solubility decreases.

137, Prorona tion of -OB is t1rSL step. Conversion of poor leaving group (-01-1) into good leaving

+

group (-OI-I 2)'

138. Meta 1 oxides lying below I Ig in electrochemical series decompose to form metal.

ZnO, MgO, CuO ~) no effect

AgP~Ag+02

'"

AgNO 3 -)0 Ag20 ------t Ag + 0 ~

139. During thunderstorm, there is formation of NO which changes to NOz and ultimately to HN03 (add-rain).

O2

N2 + 02~ NO --> N02

1'{20

~ N20 S -----'---7- HNO 3 (pH < 7)

140. Partial hydrolysis of cellulose gives the disaccharide cellobiose (CI2H22011). Cellobiose resembles maltose (which on acid catalysed hydrolysis yields two molar equivalents of D-Glucose) in every respect except one : the configuration of its glycosidic linkage.

142. Calcined gypsum does not contain CaC03 (that's sulphate one).

143. Hydrogen bonding is involved molecular force in the DNA molecule.

Crick observed the

type of hydrogen bonding

purine-purine and

Watson and

purine-pyrimidine

(instead of

pyridine-pyridine)

144. ~::~ --> ~ ~-I

C-J © (i) NaOH,A

bond (iil HNOJ ,._ v II pr (A I)

O 7Jeowp .. s

reactive (iii) AgNO,

(benzylic)

(i) NaOII,!J.

(ii) i-1N03 --'---::....___". no ppt. (iii) AgN03

H"

145. C2HSNC + Hp ---)0 HCOOf:i + C2HsNHz FOfU\IC acid

C2HSNH2 + H+ ---)0 CzHsNHj

Salt

146. t.-. E = 0, in a cyclic process.

{!.I-=-!. when n is odd 1. fen) =; 2'

11 h .

- 2: ,wen 11 IS even

and f : N -;. I, where N is the set of natural. numbers and [ is the set of integers.

Let x, /e N and both are even.

Then f(x) = I(Y)

147.

NC + CHCI, + ale KOH ~~ CH3

p-toluidlne (a carbylamine reacrion.]

148. Nylon threads are made up of polyamide some

common are

°

II

Nylon 6 H2N -fCHzTsC-OH Monomer

o 0

II \I

-HN-fCH21sC-NH-fCHi1sC-NH-

o

II

Nylon 66 HO-C-fCH21.4COOH

and H2N -f CH2r m~ Co-polymer

o 0

II II

-HN-C-+CH274G-NH-fCH27G

o °

II II·

NH-C-fCHz"14G-

149. In neopenrane, all H are equivalents.

CH3 CH3

I hv I

H.,C -C-CH3 ~ H3C -C-CH.,Cl

J j . Clz' I L

CH3 CH;>

IS!!. Liquid hydrogen and liquid oxygen are good fuel.

Again x, Y EN and both are odd.

Then

f(x) = f(y)

x-I y-l -2-"'-2 ~ x'=y

i. e., mapping is one-one.

Since each negative integer is an image of even natural number and positive integer is an image of odd natural number. So mapping is onto. Hence, mapping is one-one onto.

2. Key Idea : if ZI' Z 2 and Z3 are the VlJrtices of all equilaterai rrianglt. Then

:1. :1. :1.

ZI + 22 + z} '" ZlzZ ... z2z:l + z3z1 .

._. Origin, ~ and z2 are the vertices of an equilateral triangle, then

:1. :1.

21 + z2 '" ZjZZ

o=!> (zi + z2i =: 3z1zZ ••• (i)

Again 21' 22 are the roots of the equation z2+uz+b=O,

then ZI + 22 =' - a and ZlZ2 == b

Putting these values in Eq. (i), we have (- a)2=: 3b = a2 == 3b

3. Let Z = Ii ciO and (j) = 'i ei'~

~ Given

~ 'Pi = 1 ... (i)

11"

and arg (z) -- arg (0) = 2"

:=> (1- ~ = ~ ... (ii)

then Zft) "" IjC-iO. '2ei9 == 'lIz e ;(O-~)

From Eq. (i) and Eq. (ii), we get

Zti) = 1 Co.;"'2

1t . . 71

.. cos "2 - I Sin '2

Z~) '" - i

Note: We know that r (cos 0 + i sinO) '" r e'o

4.(1+i)X =[(l+i)(l+~)JX

1-1 (1-1)(1.+1)

=[(1 + i2~:] x ",[~~3.i.]X

1 - 1.2 2

==> o-~;r '" Cif ,,; 1 (given)

~ (if = (it",

where n is any positive integer,

;;;;:,. x =411

1, + abc = 0 :=> abc = - 1

Alternative Solution:

The vectors (1, Q, (?). (1, b, b<), (1, c. (2) me non

coplanar.

.. Their scalar triple product if. 0

I 1 a a2

i.e., I'} b b2 #-0

1. C c2

... (i)

1l is given that

1 + (1"1 i

1 + ~J~ \= 0 I .~ e~\

,

a a2 1 In (l2 a:1
b b2 I I/- b3
=> } +\b =0
c cL 1 C L c:l
c
la (12 i 1+ abel ~ a c? I
==> b b2 b 1/ =0
·c ,2 C ,2.
11 P 1
il a Q21 11 a aZ I
:=> ! ] b bZ!./_ abc 1 b b~ ~O
'1 c c~ i 11 c c21
- (1 + abc)i i a 2
(l .
o=!> b b2 ",0
1 c c:O
, ==> abc * -1. (using (i))

6. Key Idea: The s),stem of linear equations QIX + /)1)' -l- ctz::: 0

QZX + b?y '1' Czz = 0

and a3x + hi)' + C:)z '" 0

The system of linear equations has a non-zero solution, then

I} 2.a 01

]1.3b b=O 1 4c c]

J\pplyingRz -7 R2 -·R), H:{ -7 H:l-· /{l

\ I 2.a a I

o 3b-2a b-(.II=O

o 4e-2.a c-n,

0:::. (3b - 2u) (c - cr) - (4c - 2.a) (b - c) =0 0

==> 3be - :ib" -. 2a.c + 2a~ '" 4bc - 2ab - 4ac .~ 2a2

=> 4ac - 2ac = 4bc·· 2£1.b - 3h, + 3ab

=> 2ac = be + ab

On dividing by abc, weget

2 I J -il =0. + C

i. e., Cl, 11, c are in HP.

7. Key Idea: If 0:, l> be the roots of the equation axZ + bx + c + 0, then

b c

a + l> = ~ - and a~ = -.

a a

Given equation is

ax2 + bx + c == 0

Let a, !3 are the roots of this equation.

Then a + 13 == - £ and ap = .£

a a

1 1 0:2 + n2

Also a + t3 =; - + - = -."....;p:__

IX 2. 132 IX 2152

a + 13 = (0: + l3i- 2aP Calli

~ a+~=(o:;:r-;t3

~ (_ E_) = (-.bl a)2 _ 2._

a cia cia

~ _!!_ = (£)2 _ 2a

a c c

~ 2n = (£)2 +!!_

C C a

2n =!!. [£ +.£]

c c c a

2n b c 1)=c+(i

cab .

~- - -are mAP. a' b' c

abc . H

=> -'-'-barem P. c a

8. x2 - 31 x I + 2 == 0

If x » 0, then r x I = x therefore x2 - 3x + 2 = 0 ;:::) xl - 2x - x + 2 = 0 => x (x - 2) -1 (x - 2) = °

~ ex -1)(x - 2) = °

=> x =1,2

If x < 0, then I x I = - x therefore x2 + 3x + 2 == 0

;:::) x2 + 2x + x + 2 = 0 => x (x + 2)':" 1 ex + 2) = 0

;:::) (x + 1) (x + 2) = 0

;:::) x=-I,-2

Hence, four solutions are possible.

9. Key Idea : If 0. and 20: are the roots oj the equation ax2 + bx + c = 0, then 2b2 == 9ac

Since one root of the quadratic equation Ca2-5a+3)x2+(3a-l)x+2=0 is twice as

large as the other, then

=:>

2(3a -Ii' = 2x9 (a2 - 5a+ 3)

=:> 9a2 - 6a + I == 9a2 - 45a + 27

=> 45a - 6a =; 27 - 1

26 2

;:::) a= 39 ="3

Alternate Solution

The given equation is

(a2 - 5a+ 3) Xl + (3a -1) x + 2= 0

Let a and 2a. are the roots of this equation, then a + 2a = _ (3a - 1)

(a2-5a+3)

3a '" _ (3a-I)

. Ca2 - Sa + 3)

2 0:'20.= 2--(a- 5£1+ 3)

2a 2 = ----;;__:2=---...._ (a2 - Sa + 3)

=> [ - (30 - 1)]2 2

2 3(~ _ 50+ 3). (a2 - Sa + 3)

(3a _1)2 1

9 (a2 - Sa. + 3l = (a2 - Sa + 3) (30 -Ii = 9 (a2 - 50.+ 3) 9a2 + 1 - 6a = 9a2 - 4Sa + 27 4Sa - 6a = 27 - 1

39a= 26

2

a="3

10. A = [b ~ J. A 2 = [~ ~ ]

and

=>

A2=A.A=[b ~J[b ~]

=> A2=[a2+b2 ab+ba] ba+ab b2+a2

_ [a2 + b2 2nb]

- 2ab a2+b2

=> [p ~J ==[ a~::2 a22:~2]

=> 0. = a2 + b2, 1'1 '" 2ab

11. The number of choices available to him = sc" x.8C6 + sc" X 6CS

51 . B! 51 8!

= 4!1! x 6!2!+ 5!0! x 5!3!

5 8"x7 . 8x'7x6

'" x -- + 1 x --::_;_.,....:.

2 3><2 .

=5><4x7+8x7

= 140 + 56 = 196

12. The number of ways to. sit men = 5!

and the number of ways to sit women = 6C 5 X 5!

Total number of ways '" 5! x 6CS x 5!

= 5! x 6 x 5! = 6! x 51

1 ron roln
13.8= ro" ro2/1 I
ro2n I roll = 1 (o?" -1) _ (J)"{ro211- roll!) + (J)211(roR _ 0(411) "" I (1 -1) - 0+ ro2n (e" .... o") '" a

14. Ker Idea: nCr + 1 + "Cr :: n+lC,+ I-

nc,+! + "C'_1 + 2"Cr

= ncr + 1 + "Cr + "Cr-1 + "Cr = 1l+1C,+1 + l1+lCr

= "ot-lee+}

15. (I' + l)th term of (..[3 + iifS)256

i.e., T, + 1 -= 256Cr (3pS6 - r)!2 (5t/~

Th . I if 256 - r d r \.. th e terms are integra ,,1 '-2-- an "8 are no ,

positive integer.

r = 0,8,16, 24,32, ... , 256 Hence total terms are 33.

16 .. : (r+ l)th term in the expansion of (1 + Xf7 (S

¥ (¥ - 1) '" (¥- r + ] ) x'

r!

Now this term will be negative, if the last factor in Nr, is the only one negative factor.

=:). 27 -r+ 1 <o=:) 32 <1'

5 5

=:) 6.4 < r :::> least value of r is 7.

Thus first negative term will be 8th. Alternate Solution

(1 + xP/5 = 1 + 27 x + 27 (27 -1) xl

5 5 5 2\ .

+ 27 (27 _ 1) (27 _ 2) x3

5 5 5 3!

+ 27 (27 -1) (27 _ 2) (27 _ 3) X4

S 5 5 5 4\

+ 2~ (2~ _1)(2; _ 2)(2~ _ 3)(2; _ 4) ~:

+ 2; e; _1)(2; _2)(2; -3)(~; -4)e; -5) ~~ + 2; (2; _} )(2; _ 2)e; _ 3)(2; _ 4) (2; _ 5)

x (2; _ 6) ;7! + ...

Here 2; _ 6 is negative i. e" 8th term is negative in the expansion of (1 + Xl7/S•

1 1 1 '

17.}.2 - 2-3 + 3.'4- ...

= (1 -.!) - (1. - .!.) + (.! - !) -

22334:'"

1 1 1

=1-2'2+ 2'3-2.'4+'"

=2[1-~+~-~+ ... ] -1

'" 210g (1 + 1)-1 '" 2 log 2 - log e "" log 4 - log e.

4 =Ioge

18. Let lex) == Ax2 + Bx + C

.. f(l) = A + B + C

and fC-l)=A-B+C

1(1)= f(-I)

= A+B+C=A-B+C

:::> 2B = 0

B=O l(x)=Ax2 +C

Now f' (x) = 2Ax

:. /,(a) = 2Aa, J'(b) = 2Ab, /'(e) = 2Ac Also a, b, C are in AP.

., 2Aa, 2Ab, 2Ac are in AP.

=:> f '(a), 1 '(b), f '(c) are inAP.

19. If xl> X2, Xl and Yv Y2' Y3 are in GP,

then let Xl = rx1> X3 = r2xI

and Y2 = ryl' Y3 = r2Yl

with common ratio r. then the points are

2 2

(Xl' Yl)' (1'Xj, 1)'1) and (r Xl' r Yl)'

I X Y 11 Xl y, 1

Now x~ .y~ 11 = rXJ 1)'1 1

x3 Y3 1 r2Xl rZYl 1

= X1Yl I ~ ; ill

r2 r2

'" X1YI (0) = 0

(Since two column: are identical) Thus these points lie on a straight line. Alternate Solution

Let XI = a=:> X2 == 111' and X3 = a,-2 and Y '" b =:> h =, br and Y 3 = b,-'2

Let the points are A(a, b), B(ar, br) and C(ar2, b?).

New slope of AB '" beer - 11)) '" !!. ar- a

ber2 - r) b

and slope of BC '= - 2 a

aCr - r)

slope of AB '" slope of BC => MJI BC

But B is a common point.

.. A, B and C are collinear.

i.e., the points (XI' Yl ),. (X2, Y2) and (x3' Y3) lie on a straight line ..

20. AB '" a

and In~AON,

ON .LAB AN=BN

·n AN tan - Oon ON

ON '" AN cot 2! == ~ cot 2:

11 2 11

and sin ~::: AN

11 OA

OA == AN cosec 2: == ~ cosec .::

11 2 11

Sum of the radii = ON + OA

a 'It a "It

== "2 cot 11 + "2 cosec n

=!![cos ~ + ~1 ]

2 . It .. '1{

Sin n Sin n.

'" ~[.l +.c: %] = ~[.~+2~in: 1]

Sin n sin ""2it cos 211

a "It

=2 cot 211

2C 2A 3b

.21. a cos 2: + c COS 2" = "2

=> a [~ (.~b C)] + C [ s (Sb~ a)] =3i

S [s - c + s - a] 3b

·b 2

2 s [2 S .. C - a] == 3b2 2 5 [a + b + C - C - a] '" 3b2 . (a + b +e) b '" 3h2

=>

=>

. a+b+c=3b 2b;"a+ c

i.e., a,b, c are in AP.

. C JS(S-C) A ~s(s-a)

Note: C05 2 = ... '---ab and cos "2" ~.

22. Given AD == 4 and BD= DC A

B ·D

In 6 ABG tan .:: = AG

, ·3 BG

BG '" AG cot ~

c

818 BG=-x-=---

3 J3 3.J3

B

1

"'2 xBG xAD

1 8 16

=-x--·x4=-· --

2 3.../3 3../3

Since, median divide a triangle into two triangles of equal area. Therefore

Area of t. ABC == 2 x area of 6 AVB

=2x~=~

3./3 3..[3

Area of !) ADB

23. Key Idea: The range of sin-l x is [ -~, ~l

We have sin -I. x = 2 sin -1. a

=> -.:: < sin " x <.::

2 -. - 2

- 2! < 2sin-l as'::

2- 2

- ~. < sin -J a < .!:

4 -. - 4

. (It) . 'It

Sill -"4 .,; a s Sin "4

1 1

--::;a$-

.J2 12

lal.s;2- ..f2

24. Given 92 ::: tan-1 ~

"R

cth

9, H

O~_. __ 40m ~A

3

tan 9:. "'"5

AC

In 6 AOC, tan Elt '" OA

1

""'h h

.±__ =-- 40 160

Infl .AOn,

A8 h

tan (81 + 92) '" OA '" 40

taneJ +tan92. h 1 .: tan ~ tan 92 '" 40

h 3

160 +"5 h

1 __ h ,,~ '" 40 160 5

S[h + 96] It

800 - 311 40

.=;:> 200 [h + 96) '"' 800h - 3112

~ 200h + 19200 "" 800h - 3h2

=> 3h2 - 600h + 19200 = 0

::::;> 112 - 200h + 6400 == 0

=;;. (h-160)(h-40)",,0

=> 11 = 1600rh =40

.. Height of the vertical pole == 40 m,

1 25. f(x) = x + - x

f'(X)=l-~\

Put f' (x) = 0 for maxima and minima 1

1--=0

x2

x=±l

f"(x)=~-

x3

At x'" 1, f" (x) 0= + ve

., f(x)will be minimum at x = 1. Value of f(x) at x == 1 is 2.

But at x = -1 value of f(x) "" - 2

Thus, f(x) attains minimum value at x = - 1.

"

26. L f(I-)~f(I)+ f(2) + f(3) + ... +/(n)

r- 0- I

= fel) + 2/(1) + 3f(l) + ... + 71/(1) (since fex + y) = fex) + I( y)) '" (1 + 2+ 3+ ... ~- n)f{l)

== n (n + . .!2. 7 = 7n Cn + 1)

2 2

27. f(x) = x" =:_, 1(1) = 1 f'(x)=nx,,-J =>1'(1)=71

r·cx)= Tt (n -1) X"··2=> f"el)", n (n -1)

rex)= 71 (71 -1)(n - 2) 2·1

rel)= n en -1)(71 - 2) 2·1

We have

1'(1) f"(I) 1"'(1) f(l)-I! + ---z-!-~+ ...

(-Irf"(l)

+--71-"--

= 1 _ ..:!.. :: Cn - 1) _ n Cn - 1 len - 2) +

I! + 2! 3! .

(-1)" n (Ii - l)(n - 2) 2.1

+ n!

'" (1-1)" == 0

28. -_. fex) = ~ + 10g1Q (Xl - x) 4-x

For domain of fex)

::::;> x3 - X > 0

=> x (x -1)(x + 1) > 0

-y-:yy;

-1 0 1

Region is (- 1, 0) u (I, ()(J)

and 4 - x2 ~ 0

::::;> x~± 2

Region is (- 00, - 2) u (-2, 2) u (2, OC) •

:. Common region is (-1, 0) v (1, 2) u (2, (0).

[ 1 - tan 1-] [1 - sin x]

29. lim

x-.~ [1+tan1][1t-2X]3

x = ~ - h as x ~ ~ h ~ 0

.2 2'

1 - tan (% -~) (1 _ cos h) lim

11 _,_ 0 (11: It)· (2 h):l

l+tan 4-"2

2 . 2 h , h Sin '2 = lim tan -2' 8h3

'._0

h [. h]2

'" lim !. tan 2 x Sin 2, x!

h->O 4 !! x 2 .~ 4

2 2

. 1 [tan~: [Sin ~12 1

'" I;l~ 32· -h- --11- 32

"2 "2

30. lim log (3 + x) - log (3 - x) =; k

x_,_o x

Let

Applying L' Hospital's rule

[3~X+3:XJ

1 = k

lim

x_,_o

Note: L' Hospital's rule can be applied only, if a fu nction is of the form of (g) form.

31. lim fea) g(x) - fea) - g(a) f(x) + g(a) = 4

x"".o g(x)- f(x) ..

Applying L' Hospital rule

lim f(a)g'(x)-g{a)f'(x)=4

x ..... a g'(x)- f'ex)

lim

k g'Cx)- kf '(x) = 4 s' (x) - f '(x)

x ..... 0

~ k=4

32. fex) = log (x +~)

~ f( - x) '" log (- x + ~ x2 + 1)

~ f(x) + it- x)", log ex + ~X2 + 1)

+ log (- x + ~ x2 + 1)

'" log (1)'" 0 Hence f(x) is an odd function.

Note: A function is said to be an even function, if (x) '" fe-x), Otherwise it is called an odd function.

33. Key Idea: Ajim.ction is continuous in an interval [a, b1 if it is. continuous at x = a and x = band each point of interval (a, b).

Continuity at x = 0

( I. 1)

f(O-O)= lim (O-h)e- t=hf+(-hl

h ..... O

= lim (-h)e-(~-T.) == lim (-h)=O

h40 h~O

f(O+O)", lim(O+h)e-(~+T.)

h~O

~ fCO - 0) = fCO) '" f(O + 0)

Therefore f(x) is continuous for all x. Differentiability at x '" 0

-n+ ~)

Rf' (0) = lim he - 0

h ..... O h - 0

= lim Lh = 0

h ..... O e

-n-t.)

Lf '(0) = lim (- h) e - 0

h~O (-h)-O

lim eO = 1

h ..... ·O

Rf '(O)~Lf '(0)

f(x) is not differentiable at x '" o. 34.': fex) =2x3 - 9ax2 + 12a2x + 1 f '(x) '" 6x2 - 18ax + 12a2

Put f' (x) = 0

6[x2 - 3ax + 2a2] = Q

x2 - 3ax + 202 = 0 x2 - 2ax - ax + 2a2 = 6 x ex - 2a) - a ex - 2a) = 0 ex - a) (x - 2a) '" 0

~ x = a, x = 2a

Now f "(x) '" 12x -18a

at x » c

f "(x)= 12a-18a=- 6a

:. fex) will be maximum at x = a i.e., p s= a ,

at x= 2a

f "(x) = 24a -18a = 6a ., f(x)wilI be minimum at x'" 2a i.e., q= 2a.

Givenl =q

~ a2=2a~a=2

35. F(t) = r; fCt - y) g(y) dy

= I;el-Y ·ydy =et, I;e-Y ydy = l [(-ye-Y)~ - I;l (-e-Y) dy] = el [(- te-t - 0) - (e-Y)~]

= et ( _ te-r _ (e-t _ eO)] ..

= l [ - te" - e-t + 1]

= [1 - e-r (1 + t)] i =et-(1+t)

36. Key Idea:

ex f(x)dx= I: (a + b-x)f(a+ b-x)dx

Let' I '" r: x fex) dx ... (i)

~ 1= I:ca+ b - x)f(a+ b- x)dx

=> 1= I:ca+ b-x)f(x)dx

[Since given f(a + b - x) == f(x)]

=> I = (a + b) r>ex) dx - I: x f(x) dx

~ I == (a + b) I: f(x) dx - I [using Eq. (i)]

~ 2I=(~+b)I>(x)dx

~ I=(a;b) I:f(X)dx

r" sec~ dt (0 ) ,

37. lim o. ~O fonn

,,-.0 xsmx

Applying L' Hospital's rule

sec2 x2. 2x

= lim .

x .... o X cos x + Sin X

Again applying L'Hospital's rule

,. 2x. 2 sec2 x2 . tan x2 • 2x + 2sec2 x2

; 1m --------~-----------------

x 40 - x sin x + cos x + cos x .

0+ 2sec2 0 =1

0+2cosO

38. I '" J; x (1 - xtdx Letl-x~z => -dx=dz

1 = r(l- a) z"( - dz) == J; 0. - z) z"dz

I [n .. , n+ 2]:

= I (z"-z",·I)dz =:: _z z_

o n+1 0+2

o

1 1

=n+l-0+2 39. Key Idea: lim ! £ (.:.)4 = r x4dx .

n ... "" n r",1 n 0

1 + 24 + 34 + ... + n4

lim

c: , lim ! £ (.:.)4

rI-." n ,;1 n

. 3

I· 1 I' 1; (r)

- 1m - x 1m --... -

n--)ll)!l n !1~-orJ n ,.=1 n

= r1 X4 ax _ lim 1. x II x3 dx

Jo n->",n. 0

=[ x:I -o=~

d eStn x

40. dxF(x)=---x-, x>o

On integrating both sides, we get eSlnx

F(x) == J x- dx ... (i)

Also [4 ~ estn ,,' dx = [4 3x2 • e'in ,,3 dx

I X I x3

= F(k) - F(l) Let Xl = z => 3x2 dx =:. dz 64 sfn.

:::} J _e _ dz » F(k) - Fo.)

I z

Using Eq. (i)

[F(Z)]:4 = F(k) - F(l) .

:::;. F(64) - F(l) = F(k) - F(1)

:::} k = 64

41. Y =1 x-II =={_:-/i, ~:~

and y = 3 _I x I == {3 + x, x < 0 3- x, x> 0

Solving y := x-I and y = 3 - x

:::;. :::;. :::} and

x-1=3-x 2x=4 x=2 y=3-2

y=1

AB2 = (2 -If + (1 - 0)2 = 1 + 1 =2

:::} AB=.J2

and Be2 = (0 - 2/ + (3 _1)2 = 4 + 4 = 8

Be = 2./2

Area of rectangle ABCD = ,AB x Be

=./2x2J2.

= 4 sq unit

42. Given f '(x) == f(x) and f(O) = 1

Then let f (x) =:. C

Also f(x) + g(x);;; x2

g(x)=x2 -c

Again

J; I(x) g(x) dx == J; c (x2 - c) dx

== f~ (x2C - e2x) dx

== J; >c dx - J; e2"dx =[x2C - J2xCdx]~ - ~[iX]b

== [x2ex - 2 (xC - C)]~ - .! (e2 _ eO)

.. 2

[ 2_x ... ......x ..,,_x. 1 1 2

= X to -~" + 0.: lo - 2. (e -1)

== [(x~ - 2x + 2) C]~ _l e2 + l 2 2

= [(l - 2 + 2) el - (0 - 0 + 2) eO] _ ~ e2 + ~ 1 -, 1

=[e-2]-2e~+"2

123

""e-"2e -"2

43. General equation of parabola whose axis is

y2 =4a(x+h)

On differentiating with respect to x

dy dy

2y dx = 4a :=) y dx = 2a

Again differentiating, we get.

y (dy)2 + Y d2y '" 0

dx dx2

This is a differential equation whose degree and. order are 1 and 2 respectively.

-1 . dv

44. (1 + y2) + ex - etan Y) dx. = 0

2 dx -I

.:::} (l + y ) - + x = enID Y

dy

dx 1 eLan-1 y

:=)-+ -- X=---

dy 1 + y2 1 + y2

I ~l .• dy

IF=e1pJY=e l+y2 =elan-Iy

Therefore required solution is

-1 -1 e",n-1 y

X_elan y = felan y ·---dy + kl 1 + y2

xe,an -1 Y

-1 1 2 -1

xetan y = _ e tan y + k]

2

2x e,an-J y = e2 tan--I y + k

45. Let Pea, ~) be the point which is equidistant to A(al, h:t) and Reaz, b2)·

PA=PB

:=) (a - ali + 03 - bJ l = (a - azl + 03- b1.l

=> 0.2 + at + 2a1 Ct + 131. + bf - ~bl

'" a 2 + a~ - 2aa2 + 132 + bi - ~b2

=> 2(a2 - al)a + 2(b2 - h:t)!3

+ (at + b:t2 - a~ - bi) '" 0

=> (a2 -ad a + (bz - b:t)!3

1 1. 2 2 2

+ "2 (al + bi - a2 - b2) = 0

Thus the equation of locus is

((1.2 - al)x+ (b1. - h:t)y

1 (2 1.2 2 b2) 0

+"2<li+"l-a2- 2 '"

But the given equation is

(a2 - al)x + (b2 - l1)y + coo 0

1 (2 b2 2 b2

C = -"2 a1 + 1 - (12 - 2)

1 2 b2 2 2

= "2 (a2 + 2 - a1 - bi)

Alternate solution

(aI' h:t land (a2, b2)satisfy the equation. So that

al (al -a2)+ bl (bl -b2)+ c=O (i)

and a2 (al - a2) + b2 (bl - b2) + c = 0 (ii)

On adding Eqs, (i) and (ii), weget

(al + a2)(al - a2) + (b: + b2)(b] - b2) + 2c = 0

:=) 2c = - [af - ~ + b? - b~]

1 2 ~ 2 2

=> c ="2 [a2 + b2 - al - bi]

46. Co-ordinates of the vertices of a triangle are (a cost, a sin t), (b sin t, - b cos t) and (1, 0). centroid is

and

a cos t + b sin t + 1

x"". 3

3x - 1 '" a cos t + b siri t

a sin t - b cos t + 0

y== 3

... (i)

~ 3y"'asint-'-bcost .,.(ii)

Squaring and adding Eqs, 0) and (ii), we get (3x - 1/ + (3yi

== (a cos t + b sin t)2 + (a sin t - b cos t l

(3x -1/ + (3)'i = a2 + b2

47. The given equation is x2 - 2pxy - y2 == 0 .

On comparing with

ax2 + 2hxy + by2 = O,we get

a = i, b = -1, 2h "" - 2p=> h ;0 - P Equation of the bisector of angles

Xl _),2 xy

a:=-b=T

.ry

2 2 X -y

1+1

-p

2 2 2xy

x -y =---

p

... (i)

But given equation of the bisector of angles is

x2 - 2qry - y2 = 0 ... (ii)

On comparing (i) and (ii), we get 2

-·=-2q =>pq=-l p

48. Line OA makes an angle u with x-axis and OA = a, then co-ordinates of A are (a cos a, asin a).

B {<H)OS {gO'" a:}, a sin(90',_ Ct))

A(acos a:, a sin a)

Also OB .L OA, then OB makes an angle (90Q + a )with x-axis, then co-ordlnatesof B are (a cos (900 + a), a sin (90" + 0;))

'" (- a sin (:t, a cos (1) Equation of the diagonal not passing through the origin is

- • a cos CI. - a sin u

(y-asmo;)"'----, - -- (x-acosct)

- asm 0; - a ccs 0;

, sin 0; - cos u

:::;. (y - a sm o ) == . . (x - a cos 0:)

smo:+cos(X.

::::) (sin 0: + cos 0:) (y - a sin a)

= (sin ct -cosa)(x-acosa)

:::;. (sin a -r- cos c.) y - a sin a (sin a + cos a)

==- (sin u - cos u) x -acos ct (sin .u - cos u) ::::) y (sin a + cos c ) -t- x (cos o - sin «)

=U sin 0; (sin u + cos ce)

- a cos u (sin u - cos u) ==- a [sin 2 a: + sin IX cos U

- cos ct sin ct + CO$2 o.] :. y(sin ct + CGS ct) + x (cosa - sin 0.) = a

49. Key Idea : Two circles of radii Ii. and '2 respectively intersect in two distinct points, if Ii. - 1'2 < CIC 2 < 1', + 1'2 .

The equation of first circle is

ex -ll + (y - 3l ==?Centre CI (1, 3) and radius 1', ==- r and equation of second circle is

x2 + y2 _ 8x + 2y + 8 '" 0

~_:_-~

CentreC2(4, - 1) and radius r, == ~42 + 12 - 8

==,/17 - 8", 3

Two circles in tersect in two distinct points, then

rl-r2.<CIC2<fl+r2

:::;. r- 3 < 1(4 -1/ + (-1 - 3)2 < I' + 3

::::) I' - 3 < ,)9 -+ 16 < I' + 3

~ 1'-3<5<r+3

=0> r-3<SandS<r+3

~ 1'< 8 and 2< I'

2<r<8

50. The equations of diameters of a circle are

2x-3y"'S ' . .,0)

and 3x - 4y '" 7 ... (ii)

On solving Eqs, (i) and (ii), we get

x = 1, y '" -1.

r, Co-ordrnates of centre are (1, -1) and area of the circle", 'ltr2

154,., 22?-

7

1"=7

Equation of the circle is

(x - 1/ + (y + II = 72

x2 _ 2x + 1 + y2 + 2y + 1. == 49 x2 + y2 ~ 2x + 2y == 47

51. Ker Idea : Equation 0/ normal a, a point (ae ,2at) on a porabola y2 = 4ax is

Y = - tx + 2ae + bt 3.

Equation of the normal at point (bl ~, 2 bt I) on parabola is

y =-[1 x+ 2bt1 + btr

It also passes through (be 22, 2bt 2), then

. 2bt2 =-tl· br/ + 2h!"j + btI3 2t2 - 2tl = - tj (t22 _tI2)

= -tl (t2 + t1)(t2 - tl)

=0> 2= - t) (t2 + [1)

2

::::) [2+tl=--

[1

2

::::) t2c:::-tj--

tl

Note:

1. If a normal drawn at a point tj on the parabola meets the parabola again at a point f2' then

2 .

t2 = - tr t;·

2_ Three normals can be drawn at a point on a parabola.

3. Foot of the normals drawn from a point are called conormal points.

. . x2 y2

52. Key Idea: The fOCI of al1 ellipse 2" + """2 = 1 are a b

_ .' •. x2 yl

(±a.e, 0) and foCi of a hyperbola a'2 - r: lore

(±a' e', 0).

Equation of the hyperbola

x2 y2 .

-------=1

Cs2r (~r

Eccentricity is given by

b,2 .: a'2 (e,2 -1)

81 = 144 ( ,2 -1)

25 25 e

,2 9 25

e .: 1 + 16 = 16

, 5

e.:-

4

Then, foci of a hyperbola are

( 12 5 )

(± a' e", 0) == ± 5 x 4' °

'" (± 3, 0)

Equation of the ellipse is x2 y2

16 + II == 1

Foci of an ellipse are (± ae, O) "" (± 4e, 0).

But given focus of ellipse and hyperbola coincide, then

4e=3 =;> e=~ Also b2 == a2 (1 - (2)

=16(I-l~)=I6-9=7

53. Vector perpendicular to face OAB '" iii y

B

A x

z

~ .-.)., ..... ..... "" ..... A .....

= OA x OB '" (1 + 2j + k) x (2i + j + 3k)

==(5i-J-3k)

Vector perpendicular to face ABC == ~

-4--1> AA A AAA

== AD xvAC'" (i - j + 2k) x (-2i - j + k)

== (i - 5) -3 k)

Since angle between faces equals angle between their normals. Therefore

5 x 1 + (- 1) x (- 5) + (- 3) >( (- 3)

=752 + (-Ii + (- 3/ JI2 + (- 5)2 + (-3l 5+5+9 19

55 J3s == 35

a= cos-J G~)

54. Equation of sphere is

x2 + y2 + Z2 + 2x - 2y - 4z - 19 '" 0 Centre of the sphere is (~ 1, 1, 2)

and radius =~(-li + (Ii + (2/.+ 19 =55=5

Equation of plane is

x + 2y + 2 z + 7 '" °

Length of the perpendicular from centre 0 on the plane is

ON",-Ix1+Ix2+2x2+7

. ~I2 + 22 + 22

'" 12=4

3

In ~ OEN, OEl '" ON2 + NE2

==> 52 = 42 + NB2

==> NB2 == 25 -16

==> NE2 '" 9

==> NB: 3

55. The lines

and

x-2 y-3 z-4 -I-=-I-:~

x-I y-4 z-5 -k- = -2- = -1-

.. ,(ii)

".(i)

are coplanar, if

Y2 - Y1Z2 -ZJ I

ml nJ '" 0

m2 n2

IlL i !kl=o

k 2 1

==> -1 (1 + 2k)-1 (1 + k2)+ 1 (2-k):O -2k - 1 - 1 - e + 2 - k = 0 _k2 - 3k '" 0

k '" 0 or - 3

56. Equa dons of the lines are x==ay+b, z=cy+d

x-b z-d

=r :» -c-=y

x-b y-O z-d -a-=-I-=-c-

x = a' y + b' , z = c' y + d'

x - b' z- d'

-0'- = y, -c'- == y

x - b' y - 0 z - d' ~=-I-=-c'-

... (i)

and

... {li)

~

Eqs, 0) and (ii) will be perpendicular to each other, if

1112 + m1m2 + nln2 == 0

=> aa' + 1 x 1 + c x c' = 0

=> aa' + cd + 1 = 0

57. Equations of sphere and plane are

? 2 2

x: + Y + z + 4x - 2y - 6z- 155 == 0

and 12x + 4y + 32 - 327 = O.

Centre of the sphere is (- 2, 1, 3) and radius of sphere

= ~4 + 1 + 9 + 155 == ../169 == 13

Lenght of the perpendicular from centre on the plane

=1- 2x12+ 1 x4+ 3x3-3271 .J144 + 16 + 9

=; j'-111~ 3271

;; 338 = 26 13

Shortest distance between the pi ane and sphere = 26 - radius of sphere

== 26 -13 = 13

58. Consider OX, OY, OZ and Ox, Oy,Oz are two system of rectangular axes.

Equation of the plane correspondmg to OX, OY, OZ as axes is

X y Z

a + b + c == 1 ... Ci)

Similarly. equation of the plane corresponding to Ox, Oy, Oz as axes is

.:! + y + .:.. = 1 C")

a' b' c' ... 11

Length of perpendicular from origin to Ci) and (ii) must be same,

. 1 1

L e .• r=:o====

. I a~ + bI2 + c~ J1_ + 1...2 + J....2

V 1 a,2 b' c'

59. ("it + 1i + 7.? = ("it + -s + ~). ("1 + "It + 7) ~ ("1+ "ff + -~y = I ~ 12 + l-S 12 + 1"112

+ 2 (it, -s + -g ,"1 + 7. . it) => 0=12+ 22+ 32+2C;t·-s +"'ff.~ + 1.1') => 2 ("1 ' -g + It . ~ + "'l . it ) '" - 14

~ -rl> ~ ~ --t--t

a . D + Ij . c + C ' a = -7

~ ~ ~. --t _. --t-)

60.(u + v -w)·[(u - v)x(v -w)]

--t --t --t --t --t -+ -) == (u + v - w)· [u x V - U x W

--t --t -'10 ~

~vxv+vxw]

~ -) --t -* --t --t ~ -+ -) + v . u x V - v· u xW + v· V xW

-\i.1t x~ +~:ti x~-~.7 xiZ

~ ~ ~_~ --t --t --t 4 -t == U . v xw - V . U xw - w· u xv

--t -+ --t == U· v xw

Since (it Ii ~ ] = [tl ti ~] '" [~ It ~ ] '" [~ ~ ~l

'" (w: ti w: I "" [w: ~ :w J = o

Note:

--t ~ ~

If u, v, ware coplanar vectors, then

--t --t--t

[u v w) =0

61. Given

---+ ............... ~ ...... ~ ....

OA=7i-4j+7k,OB=i-6j+lOk,

--t ~ ~ A ~ A" ~

OC== - i - 3j +.4k and OD '" Si - j + 5k

AB = ~(7 _1)2 + (- 4 + 6i + (7 - lOi

:= -../36+4-;9 := ..J49 =-7

Be:= JCl + Ii + (- 6 + 3)2 + (10 - 4i := J4 + 9 + 36 := J49 == 7

CD == ~ (- 1 - 5)2 + (- 3 + 1 i + (4 - 5)2 = . ./36+ 4+ 1 =.J4l

_ DA = Jes _7)2 + (-1 + 4)2 + (S-7l = J4 + 9 + 4 = J17

Thus no option is correct.

62. Let D is mid point of Be.

A

BL-----~D------~C

-,» -,»

~ AB.j·AC

:. AD '" 2

(3i + 4 k) + (Si - 2j + 4 k) 2

8i-2j+8k 4~ ~ 4k =--2--= l-J+

-,» -,» r:-----

:. Length of AD =1 ADI =v42 + (-ll + 42

= J16 + 1 + 16 = 53

63. Total force

-7 ~A (- A~A

F = (4 i -i- j - 3K) + (3 i + j - k)

=71 -l- 2J-4k

Displacement

-7 A AA A A A

d = (5i + +J + k)- (i + 2j + 3k)

;:(41+2)-21{)

-,» -,» Total work done = F . d

= Oi + 2)- 4k) - (41 + 23.- 21<) := 28 -f 4 + 8

'" 40 unit

-)0 A A_' A A -)0 A A A

64. u = i + j, v = i - j and w = i + 2 j + 3 «,

Ii is a unit vector such that

and

Therefore, n is perpendicular to both 11 and-,) But Ii, ~ lie in the X)' plane.

~ n is a unit vector along z-axis

'-* A

Hence, I w . n I = 3

65. Median of new set remains the same as thai of the original set.

66. Given N = 15

L Xl = 2830, LX == 170

One observation 20 was replaced by 30, then 1: Xl "" 2830 - 400 + 900 = 3330

I. x;; 170 - 20 + .30 = 180

Variance, c:/ == k ;~ - C::Nxf

= 3i~_Q - (\8~r

3330 ._ 15 x 144

= ---15·--- .. -

'" 3330 - 2160 =:U 70;= 78.0

15 15

67. The probability that Mr. A selected the loosing horse'" j: x 1 = ~

5 4 5

The probability that Mr. A selected the winning

horse", 1 - ~ =: ~ .

68.0 S; peA) $ 1, 0 S PCB) S; 1, 0 S P(C) S 1 and ° S peA) + PCB) + peC):S 1 3x + 1

0:::;· "3 -Sl

] 2

=> -~SXS;3

Osl-X<l . 4 -

and

-3:S;xSI

1 - 2x 0:::;-2-$1

1 1

=> -"2::;x:5"2

3x+l I-x 1-2x and a s -3- 'I, -4-' + ~ s; 1

o s ]3- 3x $12

1 13

=> 3:::;x::;'-3

From all these conditions

1 1

.i:« x <._

3"" -2

69. Key Idea: The meaJl arid variance of a random variable in binomial distribution are np a.nd npo

and

np = 4 and npq = 2

1 1

q'" 2' but P + q -=1;::;. P <= 2

1 nX2"'4:=on==8

We have, P ex::::. r)", "Crp' q"-r

8 (1)7 (1)1

P (X == 1) = C1 '2 - '2

= 8 x..!.. =1_",_!. _ 28 2S 32

70. Key Idea : Resultant force of two forces P and Q acting at an angle ct, is given by

R == ~p2 + Q2 + 2PQ cos a

If a is the angle between P and Q, then

R2 =p2 +Q2 + 2PQ cos n ... (i)

---) ---t

If Q is doubled, then R is doubled

:::> 4R 2 = p2 + 4Q2 + 4PQ cos a. ... (ii)

Since,

Again if the direction of Q is reversed, then It is again doubled

=:> 4R2 = p2 + (- Qi + 2P (-Q) cos ct

4Rz =p2 + Q2 -'2PQ cos a ... (iii)

Adding (i) and (iii)

'SR2=2p2+2Q2 C')

•. , IV

Multiplying by 2 in (iii) and adding (ii) 12R2 == 3P2 + 6Q2

:=0 4R2 =p2 + 2Q2 ... (v)

Subtracting (v) from (iv), we get

R2 = p2

Putting in (v)

4R2 =R2 + 2Q2

=:> 3R2 = 2Q2

p2 Q2 R2 2'''''3''''2

=- p2:Q2:R2=2:3:2

71. Since RI and Rz respectively be the maximum ranges up and down an inclined plane, then

u2

R = -,.--.....,..-,----

1 g(l + sin (3)'

Uz

R = --".-..,.......,,.,.

2 g(l - sin (3).'

U2

R=-,

g

No _!. - _!_ _ g(1 + sin )3) g(1 - sin P)

w'R + R - 2 + 2

1 2 U U

_ g[l + sin p + 1 - sin 13]

- u2

2g 2.

=~=R

1 1 2

=>-+-=-

RI R2. R

1 1 1 ,

=>R'R'RaremAP

1 2

=- Rj, R, Rz are in HI',

72. LetAB = d.

AM is the perpendicular distance from A to B. In/!"ABM,

",_N

sin 0= AM AB

AM == d sin 0 Moment of couple,

---) --,). ---)

G == p. AM == P d sin 0 ... (i)

---)

Where P is turned a right angle, then

Moment of couple,

---) ---) ---+

H = P . d sin (900 + 0) = P d cos e .. , (ii)

---)

When P is turned through an angle c, then

Moment of couple

-t

'" P - d sin ((I. + 0)

---)

'" p . d [sin Ct cos f) + cos a. sin 8]

---) ---)

'" (P . d cos fl) sin ct + (P . d sin f») COS'(l

-t -t

'" G cos a. -I- H sin a. [Using (i) and Cii)]

73. We have

R 2 = u2 + f 2(,2+ 2uft cos (1800 - ex) R2 = u2 + f2t2 - 2uf! cos a

2 2:1

V = ~ __ "!'_L~ __ :_~~.[t__c_~~ a.

Let

,

"

I

, , , ,

I

,

, , ,

dV 2

dt ::: 0 + 2/ t - 2ui cos a

d2V 2

-=2/:::+ ve

dt2

i. e., velocity will be least after a time.

dV 2

dt :: 0 '" 2f t - 2uf cos a

==> 2f2 t '" 2uf cos a

2ui cos a

==> t::-'-----'

2f2

u cos a

=:»:

74. When stone is projected horizontally, then u

R""ucosOxt A

and 11::: usin Oxt + ~t2 J

=:- R "" ut and h = 2. gt h

~ R =ut an_dt =f{ 1

=> R '" u pgh ... 0) q~, "'-~~-==-A-=--=--=----~-'

When stone is projected at an angle of e to the horizontal, then'

R=ucosflxt ... (ii)

1 2

and h '" - u sin e x t + 2. gr , ... (iii)

USing Eq, (I) in Eq. (ii), we get

u.J¥'=ucosext

t == c'.015 ,r..,. (2h 2gh

=:- cr Vg ... (iv)

u=o -f __ r ""='O
A v B
1.01 x, C x2--'---i and tJ + t2 == t

We have from A toC

)12 =u2 + 2fxl

v2 == 0 + 2fxl

and =>

)12

Xl "" 2/

)I"'U+ ft v", 0 + ftl V

tl =J

From C to B, )12 '" u2 + 2ft V,2 == v2, - 2rxi 0,=)12 - 2rx2

and v' = v - rt 2

=> O=v-Itz

On adding Eqs, (i) and (iii?

Xl + Xl = v; (} + ~)

2s 2 (1 1)

=V 1+-':

On adding Eqs. (il) and (iv)

=> tl+tZ=V(Y+n

=> t ==)1 (1 + n

t2 =v2 (j + ~r

... 0)

... (Ii)

... (iii)

... (iv)

... (v)

... (vi)

Вам также может понравиться